Final extra tito

Lakukan tugas rumah & ujian kamu dengan baik sekarang menggunakan Quizwiz!

Gil and Tina are newlyweds who live in a community property state. Assuming no titling changes were made, which of the following assets would be separate property? A parcel of land owned by Gil prior to marriage An antique desk Tina inherits from her aunt during the marriage A money market account gifted to Gil before marriage

The assets in statements I, II, and III are all separate property. LO 1.3.3

George and Claire are married and have four children. If they elect gift splitting, what is the maximum amount of total gifts they can make to their children in 2020 and have the gifts be covered by their annual exclusions? A) $120,000 B) $60,000 C) $15,000 D) $30,000

If George and Claire elect gift splitting, they can give up to $30,000 to each child (a total of $120,000) and have the gifts be covered by the annual exclusion. LO 2.2.2

Which of the following forms of ownership may be held by spouses only? Tenancy in common. Joint tenancy with right of survivorship. Tenancy by the entirety. Community property.

answer 3 and 4 Community property and tenancy by the entirety are available only to spouses. LO 1.3.1

Which of the following is NOT an example of a qualified retirement plan? A) New comparability plan B) Section 403(b) plan C) Employee stock ownership plan (ESOP) D) Section 401(k) plan

A Section 403(b) plan is a tax-advantaged plan but not an ERISA-qualified retirement plan. While tax-advantaged plans are very similar to qualified plans, there are some minor differences. For example, a tax-advantaged plan is not allowed to have NUA treatment. NUA is covered later in the course. They are also not allowed to offer 10-year forward averaging or special pre-1974 capital gains treatment. Tax-advantaged plans also have less restrictive nondiscrimination rules. Otherwise, they are very similar to qualified plans. LO 1.2.1

Which of the following statements regarding the safe harbor rules for Section 401(k) plans is CORRECT? The employer can avoid ACP and ADP testing if it matches 100% up to 4% of compensation for nonhighly compensated employees. The employer can avoid ACP and ADP testing if it makes contributions of 3% or more of compensation for all employees who are eligible to participate in the plan, whether or not the employee chooses to participate. To meet the safe harbor requirements, the matching and non-elective contributions must be immediately 100% vested. The employer must provide notice to each eligible employee about rights and obligations under the plan.

All of these statements regarding Section 401(k) plan safe harbor rules are correct. LO 3.3.2

ABC Corporation is a closely held company that wants to establish a qualified retirement plan for its employees. Also, the company wants to improve the marketability of its stock and give the employees an ownership stake in the company. Which of the following plans would help ABC meet all of these objectives? A) Employee stock ownership plan (ESOP) B) SIMPLE 401(k) C) Keogh (self-employed) plan D) Target benefit pension plan

An ESOP could help ABC meet all of these objectives. LO 3.2.2

Carol has an additional retirement need of $30,000 annually in today's dollars. She will retire in 15 years and projects a retirement period of 20 years. Carol believes she can achieve a 6% after-tax rate of return and is assuming a 4% annual rate of inflation. Using the level payment approach, how much will Carol need to save in a single annual payment at the end of each year to fund her retirement need? A) $38,973.65 B) $36,767.56 C) $34,044.67 D) $30,000.00

Carol will need to make 15 level annual payments of $38,973.65 at the end of each year. Here is how to work this problem using the regular TVM keystrokes. What will it take to have $30,000 of today's purchasing power in 15 years? $30,000, +/-, PV 4, I 15, N 0, PMT FV Answer: $54,028.3052 How much needs to be in the account at retirement to fund 20 years of serial payments at the first of each year? +/-, +/-, PMT (Hitting the +/- button twice before PMT allows the entire FV of 54,028.3052 to be entered as the PMT quickly and to nine decimal places, The second +/- makes the value positive again) Shift, MAR (To get into the Begin Mode) 0, FV [(1.06 ÷ 1.04) -1] × 100 = 1.9231, I 20, N PV Answer: -$907,149.5440 How much needs to be saved at the end of each year to have $907,149.5440? +/-, $907,149.5440, FV (Hitting the +/- here allows the calculator to take the answer and turns the number positive) 15, N 6, I 0, PV Shift MAR (To be in the End Mode.) PMT Answer: $38,973.65 LO 8.3.2

How is an age-weighted profit-sharing plan similar to a traditional defined benefit pension plan? A) Minimum vesting schedules are more liberal than in other types of plans. B) Employer contributions are flexible from one year to another and, if resources are not available, the employer may choose not to contribute to the plan. C) Retirement benefits are determined by the participant's final account balance. D) Contribution allocations to older participants may be maximized, while allocations to younger participants may be minimized.

Contribution formulas for both age-weighted profit-sharing plans and traditional defined benefit pension plans favor older employees. Retirement benefits are determined by the participant's final account balance in defined contribution plans. Employer contributions can be flexible in an age-weighted profit-sharing plan, but not in a defined benefit pension plan. Vesting is not more liberal in these plans than in other types of plans. LO LO 3.2.1

Paul estimates he will need a $75,000 annual income in today's dollars when he retires 10 years from now. He assumes a 3% annual rate of inflation, a 5% after-tax rate of return on his investments, and a 20-year retirement period. What lump-sum amount should Paul have accumulated over the next 10 years to support his retirement income need? A) $1,689,612 B) $2,015,880 C) $1,657,429 D) $1,257,227

His total retirement fund needed to support his standard of living is $1,689,612, calculated as follows: The client's first-year retirement income need is $100,794. PV = -$75,000 i = 3 n = 10 FV = $100,794 The total capital required to support this need for 20 years is $1,689,612. In BEGIN mode (the client will make annual withdrawals at the beginning of each year) PMT = $100,794 n = 20 i = 1.9417 [(1.05 ÷ 1.03) - 1] × 100 PVAD = -$1,689,612 LO 8.3.2

Susan makes $400,000 working for Great Grapes, Inc. She defers 4% into the 401(k) and receives the 4% match. How much will go into her account in 2020? A) $19,500 B) $32,000 C) $22,800 D) $16,000

Only the first $285,000 of compensation may be used to determine contributions to qualified retirement plans in 2020. Thus, she contributes 4% of $285,000 in 2020. This amount is matched, so $285,000 × 0.08 = $22,800. LO 1.3.2

With an integrated defined contribution plan, what is the maximum permitted disparity? A) For an integrated defined contribution plan, the permitted disparity is the lower of the base amount, or 5.7%. B) The maximum permitted disparity is 25%, so if the base benefit percentage is greater than 25% then the permitted disparity would be capped at 25%. C) In an integrated defined contribution plan, if the base contribution percentage is 5% then the permitted disparity is 5.7%. D) The permitted disparity is 0.75% per year for up to 35 years.

The permitted disparity is the lower of the base amount, or 5.7%. Thus, the maximum permitted disparity is 5.7% for integrated defined contribution plans. The number 5.7% is the percentage of an employee's compensation that goes toward his Social Security retirement benefit for compensation below the taxable wage base ($137,700 in 2020). LO 1.3.3

Which of the following investments would be the least suitable for a qualified retirement plan? A) Municipal bond fund B) Equity mutual fund C) Guaranteed investment contract (GIC) D) Real estate investment trust (REIT)

The tax-exempt nature of qualified retirement plans must be considered in determining if a specific type of investment is appropriate. Municipal bonds are tax-free investments. Because a qualified plan is already tax exempt, there would be no reason to utilize them in plan assets. LO 8.2.2

Northwest Instruments Corp. made matching contributions to its SIMPLE 401(k) in the last three years. Assume all eligible employees earn at least the maximum includible compensation limit and all defer the maximum amount allowed. Due to extensive capital expenses anticipated this year, the company is considering how to reduce expenses. It will not be able to continue to make the 3% matching contribution and has called you to discuss their options. Which of the following could you recommend? A) Because they have satisfied the 3% matching contribution for three years, Northwest Instruments Corp. could reduce the matching contribution to 1%. B) With SIMPLE 401(k) plans, employers who begin using the 3% matching contribution do not have any option available to modify the company's contribution. C) By providing adequate notice, Northwest Instruments Corp. could move to the 2% non-elective contribution this year, although the savings would be minimal. D) Employer contributions under a SIMPLE plan are discretionary, and Northwest Instruments Corp. could provide notice that they will not provide any contribution this year.

Under a SIMPLE 401(k), an employer does not have the option to change the percentage used for the matching contribution. However, the employer can use the 2% non-elective contribution instead of the 3% matching contribution. Under the scenario described, however, the savings would be minimal. LO 3.3.3

Ellen participates in a SIMPLE 401(k) maintained by her employer. If she has completed two years of service, to what extent is she vested in the employer contributions to her account? A) 0% B) 40% C) 20% D) 100%

Vesting schedules are not permitted in SIMPLE 401(k)s. Employees are always 100% vested in employer contributions. LO 3.3.3

Which of the following events should trigger a recalculation of the retirement needs analysis? Marriage Employment change Change in FICA rate Significant income change

answers 1, 2, and 4 The retirement needs analysis should be recalculated any time there is a change in the client's life such as marriage, divorce, births, deaths, employment and income, or health. Temporary changes in tax laws will not impact the retirement needs over the long term. LO 8.3.1

In addition to meeting the financial needs and resources tests for hardship withdrawals, money may only be withdrawn from profit-sharing plan accounts for the following reasons: Purchase of a primary residence Payment of unreimbursed medical expenses Payment necessary to prevent foreclosure on the participant's primary residence Payment of higher education expenses for the participant, spouse, or dependent children

All of the statements regarding hardship withdrawal requirements are correct. LO 3.1.1

ERISA requirements for qualified plans include A) coverage and vesting. B) all of these. C) participation and fiduciary requirements. D) reporting and disclosure.

All of these are ERISA requirements for qualified plans. LO 1.1.1

Qualified retirement plans are which of these? They are subject to ERISA requirements. They offer tax-deferred earnings to employees. They can discriminate in favor of highly compensated employees. They provide a deferred tax deduction for employer funding.

Answer 1 and 2 Statements I and II are correct. Qualified retirement plans are subject to ERISA requirements and provide tax deferral on investment earnings for employees. While qualified plans in general can provide different levels of benefits to different classes of employees, qualified plans cannot "discriminate in favor of highly compensated employees" in the sense that there is a legal limit to the amount of the difference. As long as the difference is inside the legal limits, the plan is not discriminatory (by definition). Qualified retirement plans provide an immediate tax deduction on employer contributions—not a deferred tax deduction, like a nonqualified deferred compensation plan. LO 1.2.1

Which of the following statements regarding a top-heavy plan is CORRECT? A top-heavy plan is one that provides more than 50% of its aggregate accrued benefits or account balances to key employees. A top-heavy defined pension benefit plan must provide a minimum benefit accrual of 2% multiplied by the number of years of service (up to 20%). For a top-heavy defined contribution plan, the employer must make a minimal contribution of 3% of annual covered compensation for each eligible non-key employee. If the contribution percentage for key employees is less than 3%, the contribution percentage to non-key employees can be equal to the key employees' percentage. A top-heavy defined benefit pension plan must provide accelerated vesting.

Answer 2, 3, and 4 Only Statement I is incorrect. A top-heavy plan is one that provides more than 60% of its aggregate accrued benefits or account balances to key employees. LO 1.3.1

Which of the following plans is a cross-tested plan? New comparability plan Employee stock ownership plan (ESOP) Age-based profit-sharing plan Stock bonus plan

Answwers 1 ans 3 Of the plans listed, only the new comparability plan and the age-based profit-sharing plan are cross-tested plans. Cross-testing means a defined contribution plan is tested for nondiscrimination based on benefits rather than contributions. LO LO 3.2.1

Which of the following is an example of a qualified retirement plan? A) Section 401(k) plan B) SEP plan C) Deferred compensation plan D) Section 403(b) plan

A Section 401(k) plan is a qualified plan. 403(b) plans and SEP plans are tax-advantaged plans, but are not ERISA-qualified retirement plans. A deferred compensation plan is a nonqualified plan. While tax-advantaged plans are very similar to qualified plans, there are some minor differences. LO 1.2.1

Which of the following retirement plans can be adopted only by private, tax-exempt organizations and state or local governments? A) Section 457 plan B) Section 403(b) plan C) Stock bonus plans D) ESOP

A Section 457 plan can be adopted only by private, tax-exempt organizations and state and local government entities. Section 403(b) plans may be adopted by Section 501(c)(3) nonprofit organizations, and ESOPs and stock bonus plans may be adopted by corporations. LO 8.1.1

Which of the following is a type of trust in which the grantor has not completed a transfer for gift tax purposes, and has retained the power to rescind the trust? A) Complex B) Revocable C) Simple D) Irrevocable

A revocable trust is a trust in which the grantor retains access to the income and property of the trust. LO 2.2.1

Desiree just met with her financial adviser and discovered that her estate substantially exceeds the estate tax exclusion amount. She would like to reduce her potential taxable estate without giving up control of her property during her life. Which of the following premortem planning techniques would achieve Desiree's goals? A) Desiree should establish and fund a revocable living trust. B) Desiree should make a testamentary transfer of property that will qualify for the charitable deduction. C) Desiree should make lifetime gifts up to the annual exclusion amount. D) Desiree should reduce or eliminate property that is difficult to appraise or administer by sale.

A testamentary transfer of property that will qualify for the charitable deduction is a method to reduce the potential taxable estate. Making lifetime gifts up to the annual exclusion amount is a method to reduce the taxable estate without incurring gift tax. However, Desiree does not want to give up control of her property before death. A revocable living trust allows her to retain control, but does nothing to reduce her estate tax liability. Reducing or eliminating property that is difficult to appraise or administer by sale will reduce the administrative fees of an estate, but it will not necessarily reduce the potential taxable estate. Also, she does not want to give up control of her assets. LO 5.3.3

Which of the following statements regarding the federal gift tax return IRS Form 709 is(are) CORRECT? For a calendar-year taxpayer, an extension of time for filing IRS Form 1040 also extends the time for filing IRS Form 709. George gives $5,000 of separate property to his son. If Mary, George's spouse, elects to split the gift with George, they must file a gift tax return. George and Mary give $20,000 of community property to their son. No gift tax return need be filed. An extension of time for filing the gift tax return does not extend the time for payment of the gift tax.

All of the statements are correct. LO 2.1.1

Which of the following are effective method(s) of limiting or avoiding federal estate taxes? Use the annual gift tax exclusion Create an irrevocable life insurance trust Use qualified transfers to pay tuition and medical expenses directly to the provider Use the unlimited marital deduction

All of these items will reduce the gross estate or the taxable estate. LO 5.3.2

Which of the following statements concerning gifts of appreciated property are CORRECT? The donor's holding period carries over to the donee. Generally, the donor's basis carries over to the donee. If the donor paid gift tax on the gift, the donee's basis is increased by a portion of the gift tax paid.

All of these statements are correct. LO 2.1.2

Which of the following types of gifts can be made without incurring federal gift tax? Tuition payments made directly to an educational institution Payments for health care made directly to a medical care provider Transfers to a political organization

All of these statements are correct. LO 2.1.3

Which of the following statements regarding QTIP trusts are CORRECT? QTIP trusts allow a terminable interest to be left to the surviving spouse and still qualify for the estate tax marital deduction. Assets in a QTIP trust are usually included in the gross estate of the second spouse to die. QTIP trusts are useful when the first spouse to die has children from a prior marriage.

All of these statements are correct. The QTIP is included in the estate of the second spouse to die at the same percentage as the first spouse took as a marital deduction. If 100% of the QTIP was taken as a marital deduction when the first spouse dies, then 100% of the QTIP is included in the second spouse to die's gross estate. If the first spouse's estate only took a martial deduction for 70% of the QTIP, then only 70% of the QTIP's value when the second spouse to die passes away is included in the estate of the second spouse to die. LO 2.2.1

Which of the following transfers are gifts for purposes of the gift tax statutes? Kurt creates an irrevocable trust providing that his son is to receive income for life and his grandson the remainder at his son's death. Kurt purchases real property and has the title conveyed to himself and to his brother as joint tenants. Kurt creates an irrevocable trust giving income for life to his spouse and providing that upon her death the corpus is to be distributed to his daughter. Kurt purchases a U.S. savings bond made payable to himself and his spouse. The spouse later surrenders the bond for cash to be used for her benefit.

All of these transfers are gifts for purposes of the gift tax statutes. Statement IV falls under the gift tax statutes, but the unlimited marital deduction may be utilized to offset any possible gift tax due. LO 2.1.2

Which of the following statements regarding gift splitting are CORRECT? Gift splitting is permitted only between spouses. If gift splitting is elected, gift splitting applies to all gifts in a given year. Gift splitting doubles the annual exclusion for gifts of a present interest. Gift splitting requires the filing of a gift tax return.

All the statements are correct. LO 2.1.1

Jerry wants to establish a qualified plan for his business to provide employees of the company with the ability to save for retirement. Which of the following plans is a qualified plan? Profit-sharing plan Simplified employee pension (SEP) plan SIMPLE IRA Section 457 plan

Answer 1 only Of the plans listed, only the profit-sharing plan is a qualified plan. The SIMPLE IRA and the SEP plan are tax-advantaged plans, and the Section 457 plan is a nonqualified plan. LO 1.2.1

Which of the following statements regarding the attributes of property owned solely by one person is CORRECT? The maximum ownership interest a person may have is known as fee simple or absolute ownership. A lease is an example of a term for years.

Answer both

In which of the following situations must the donor file a gift tax return (IRS Form 709)? A) A father gives his son $15,000 in cash and his spouse agrees to gift splitting. B) A man pays $100,000 directly to a hospital to pay the medical expenses of his best friend. C) An uncle gives his niece $5,000 in cash. D) A husband gives his wife a house with a fair market value of $3 million.

Any split gift election requires the filing of a gift tax return, even when there is no tax due, unless the gift is community property and less than twice the annual exclusion amount. LO 2.1.1

Diane and Clara are spouses. They own their home as a joint tenancy with right of survivorship (JTWROS). They both worked during their marriage, and Diane contributed 30% of the purchase price for the home, while Clara contributed 70%. The home is currently valued at $1 million. If Clara dies, what amount will be included in her gross estate for estate tax purposes? A) $700,000 B) $500,000 C) $1,000,000 D) $0

Because Diane and Clara are spouses, 50% of the value of any property they own as JTWROS is included in the gross estate of the first spouse to die, regardless of how much each spouse contributed to the acquisition of the property. LO 5.1.2

Jose and Alfonso are brothers. They own a rental property as joint tenants with right of survivorship. The current value of the property is $1 million. Jose can prove that he contributed 80% of the acquisition price for the property. If Alfonso dies, what amount will be included in his gross estate for estate tax purposes? A) $500,000 B) $0 C) $200,000 D) $1,000,000

Because Jose and Alfonso are not spouses, the consideration furnished rule determines how much of the property is included in the gross estate of the first joint tenant to die. Because Jose can prove he contributed 80% of the acquisition price, only 20% ($200,000) is included in Alfonso's gross estate. LO 5.2.2

Which of the following is considered to be a gift for federal gift tax purposes that qualifies for the gift tax annual exclusion? A) A gift to a political organization for its use B) Property settlement transfers pursuant to a written divorce agreement C) A payment of cash to a favorite nephew to assist with his college tuition D) A transfer of property to fund a revocable trust

Because it's not made directly to an educational institution (which would be excludible as a qualified transfer), the payment of cash to a favorite nephew to assist with tuition is both a gift and one that qualifies as a present interest for purposes of the gift tax annual exclusion. The gift to a political organization is a gift, but there is no gift tax liability. The transfer of property to fund a revocable trust is an incomplete transfer; gift taxes do not apply. A property settlement transfer that is a part of a written divorce agreement or decree between divorcing spouses is not considered a gift. LO 2.2.2

Sam and Sue paid $100,000 for their home five years ago. Its fair market value was $150,000 when Sam died. What was Sue's basis in the home after Sam's death if the home was held as community property? (CFP® Certification Examination, released 01/99) A) $150,000. B) $75,000. C) $100,000. D) $125,000.

Both halves of community property receive a stepped-up basis equal to the fair market value at the death of the first spouse. LO 1.3.3

Christine transfers property worth $15 million into a grantor retained interest trust (GRIT). She retains the right to receive income from the trust for a term of 15 years. At the end of the 15-year term, the trust property will pass to her children. Christine's GRIT does not comply with the zero valuation rules contained in Chapter 14 of the Internal Revenue Code. Which of the following statements regarding the gift tax consequences of Christine's GRIT is CORRECT? Christine's retained income interest will be valued at zero for gift tax purposes. The entire $15 million in property transferred to the GRIT is considered to be a gift for gift tax purposes.

Both of these statements are correct. LO 2.1.3

Two years before he died, Alberto gave each of his three children a gift of $100,000. Which of the following statements regarding these gifts is CORRECT? The gifts are not included in Alberto's gross estate. The gifts are included in Alberto's estate tax calculation as adjusted taxable gifts (ATGs).

Both of these statements are correct. LO 5.2.1

Which of the following statements about a life estate in real estate is CORRECT? A life estate is a partial interest in property giving someone the right to possess and enjoy the property for an individual's lifetime. A life estate may be for a term for years entitling the owner to posses or enjoy the property for a fixed period of time.

Both statements are correct. LO 1.3.1

Which of the following statements regarding the filing of an estate tax return is CORRECT? An estate tax return must be filed if the value of the gross estate plus adjusted taxable gifts on the date of death exceeds $11,580,000 in 2020. The estate tax return is due nine months after the decedent's date of death, although a six-month extension of time to file may be requested.

Both statements are correct. LO 5.1.1

Which of the following describe differences between a tax-advantaged retirement plan and a qualified plan? IRA-funded employer-sponsored tax-advantaged plans may not incorporate loan provisions. Employer stock distributions from a tax-advantaged plan do not benefit from NUA tax treatment.

Both statements are correct. IRA-funded employer-sponsored tax-advantaged plans are SEPs, SARSEPs, and SIMPLE IRAs. LO 1.2.1

Which of the following intrafamily property transfers are subject to the special zero valuation rules under Chapter 14? Corporate recapitalizations Partnership capital freezes

Both statements are correct; corporate recapitalizations and partnership capital freezes are both subject to the special zero valuation rules under Chapter 14. LO 2.1.2

A married couple resides in a community property state. Their community property consists of real property with an adjusted basis of $90,000 and a fair market value of $300,000, as well as other property with an adjusted basis of $50,000 and a fair market value of $20,000. Spouse 1 died this year and left his entire estate to Spouse 2. What is Spouse 2's adjusted basis in the real property and other property after Spouse 1's death? RealOther A) $195,000$50,000 B) $90,000$50,000 C) $195,000$35,000 D) $300,000$20,000

Both the decedent's and surviving spouse's shares of the community property receive a new basis equal to the fair market value on the date of the decedent's death. The real property's basis is stepped up. Notice that the other property's basis is less than its original adjusted basis, so getting the date of death basis actually results in a stepped-down basis for the other property. LO 5.1.2

Which of the following characteristics is common to both the federal gift tax and the federal estate tax? A) The property receives a stepped-up income tax basis in the hands of the recipient of the transfer. B) The ability of one spouse to assume half of the transfers made by the other spouse C) The tax-exclusive nature of the tax D) The ability to shield qualifying transfers to a qualified charity from the tax

Both the gift tax and estate tax allow a charitable deduction for qualifying transfers. The gift tax is tax exclusive, while the estate tax is tax inclusive. Only lifetime transfers (gifts) can be split by spouses. A stepped-up income tax basis is awarded only for property subjected to the estate tax. LO 5.1.1

In 2020, George decided to begin a program of lifetime giving to his five grandchildren and three great- grandchildren. He wants to control the amount of annual gifts to avoid the imposition of federal gift tax, and he does not desire to use any of his or his spouse's applicable credit amount. However, his spouse is willing to split each gift over a period of 10 years. Over the 10-year period, George can give a total amount of gifts (ignoring future indexing of the annual exclusion), including the gift splitting, of A) $300,000. B) $1,200,000. C) $2,400,000. D) $150,000.

Calculate as follows: $15,000 × 8 × 10 × 2 = $2,400,000. Each spouse can gift $15,000 (for 2020) per donee without resulting in a taxable gift. Using gift splitting over the 10-year period, they can gift a total of $2,400,000. LO 2.3.1

Richard participates in a defined benefit pension plan at his place of employment. His projected monthly benefit under the plan is $1,000. If the plan provides life insurance for Richard, the death benefit payable under the policy is limited to A) $10,000. B) $1,000. C) $100,000. D) $25,000.

Defined benefit pension plans use the 100 times test for determining whether they comply with the incidental benefit rules. Under this test, the death benefit cannot exceed 100 times the participant's projected monthly benefit. LO 8.2.2

ERISA requires reporting and disclosure of plan information to all of the following except A) the Internal Revenue Service (IRS). B) plan sponsors. C) plan participants. D) the Department of Labor (DOL).

ERISA requires reporting and disclosure of plan information by plan sponsors to the IRS, DOL, Pension Benefit Guaranty Corporation (PBGC), and plan participants. LO 1.1.1

All of the following retirement plans permit employees to make elective deferrals except A) SIMPLE 401(k)s. B) SEP plans. C) profit-sharing plans with Section 401(k) provisions. D) Section 401(k) plans.

Employee elective deferrals are permitted in Section 401(k) plans, profit-sharing plans with a Section 401(k) provisions, and SIMPLEs, but not in SEP plans. LO 8.1.1

Sarah and Jenny, a married couple, own real estate as JTWROS, and Sarah is contemplating a transfer of her interest to Jenny. Assuming the transfer occurs, which of the following statements are CORRECT? A taxable gift occurs. The transfer is eligible for the marital deduction.

answer 2 only Only Statement II is correct. A taxable gift has not occurred because of the marital deduction. LO 2.2.1

What is the maximum gift that Bob and Stan, a married couple, can give to one donee in 2020 without paying any gift tax, assuming they have not made any previous taxable gifts? A) $23,190,000 B) $30,000 C) $22,800,000 D) $11,430,000

For 2020, the answer is $23,160,000: ($11,580,000 applicable exclusion amount × 2 donors) + ($15,000 annual exclusion × 2 donors). LO 2.1.1

Phillipe established a revocable living trust naming himself, his spouse, and his two children as income beneficiaries, and his grandchildren as remainder beneficiaries. He funded the trust with $300,000. What is the value of this trust for gift tax purposes? A) $0 B) $100,000 C) $250,000 D) $50,000

Funding a revocable living trust has no effect on gift taxes as no gifts are made to oneself. LO 2.1.1

Which of the following is NOT an advantage of making a lifetime gift? A) It provides personal satisfaction. B) It shifts income from the gifted asset to the donee. C) It diminishes the size of the gross estate. D) It excludes gift tax paid on the gift from the donor's gross estate regardless of how soon the donor dies after making the gift.

Gift tax taxes paid out of pocket are excluded from the donor's gross estate only if the donor's date of death is not within three years of the gift. In order for a donee to pay gift taxes out of pocket (instead of using the applicable gift tax credit), the donor must have already made adjusted taxable gifts of more than the exemption equivalent. LO 2.1.1

Which of the following statements regarding gifts is NOT correct? A) There may be income tax incentives for making an inter vivos gift, which can involve moving taxable income from a high-bracket donor to a lower-bracket donee. B) When a gift is made during the donor's lifetime, any appreciation accruing between the time of the gift and the date of the donor's death escapes the estate tax. C) If a gift is made within four years prior to the donor's death, the amount of any gift tax paid on the transfer is included in the donor's gross estate. D) An individual can give up to $15,000 gift tax free every year (for 2020) to an unlimited number of donees.

Gift taxes paid on gifts made within three years prior to death are included in the donor's gross estate. LO 2.1.1

If a defined benefit pension plan is determined to be top heavy, what is one practical significance of this determination? A) One of two maximum contribution and benefit formulas must be used. B) Different coverage requirements and nondiscrimination tests apply. C) Different eligibility requirements come into effect. D) One of two accelerated vesting schedules must be used.

If a defined benefit pension plan is top heavy, one of two accelerated vesting schedules must be used: either 100% vesting after three years of service or graded two- to six-year vesting. The same eligibility and nondiscrimination tests apply as for qualified plans that are not top heavy. LO 1.3.2

Some financial advisers recommend hiring an appraiser and filing an informational gift tax return of certain property, even when the donor believes that the property has a value that is less than the allowable annual exclusion amount. Why is this good advice? A) If a return is not filed, but the property is appraised, the IRS cannot contest the value of the gift. B) If a return is not filed when a gift is made, the IRS can contest the value of the gift at any future time. C) If the property is appraised and a return is filed, the IRS has only one year to contest the value of the gift. D) If the property is appraised and a return is filed, the IRS cannot contest the value of the gift.

If a gift tax return is filed, the IRS can only contest the valuation of the gift disclosed on the return within three years. LO 2.1.1

Which of the following statements regarding the concept of gift splitting in federal gift tax law is CORRECT? A) Gifts of community property require a gift splitting election. B) Spouses can elect to split some gifts but not split other gifts made within the same calendar year. C) The gift splitting election may be made by any related party who joins in the making of the gift. D) In non-community property states, the donor spouse must file a gift tax return even if the split gift value is less than the annual exclusion.

If a married couple elects to split gifts, a gift tax return must be filed by the donor spouse even if the split brings the total gift to less than the annual exclusion amount. One exception to this is where the married couple lives in a community property state. In that instance, each spouse is considered to already own a one-half interest in the property gifted (meaning that the filing of a return to indicate spousal consent is not required). The purpose of filing the gift tax return, even though the gift is less than the annual exclusion for either spouse, is to document the gift splitting to the IRS. LO 2.1.1

Scott is the fiduciary of the BSB retirement plan. The entity responsible for monitoring his actions as a fiduciary is A) the ERISA. B) the DOL. C) the SPD. D) the PBGC

The Department of Labor (DOL) governs the actions of plan fiduciaries and ensures compliance with the ERISA plan reporting and disclosure requirements. LO 1.1.1

Joellen dies on March 13. Her gross estate includes publicly traded stock that has a market value of $6 million on Joellen's date of death. Joellen's estate sells the stock for $5.7 million on May 2. The same stock has a market value of $5.5 million on September 13. If Joellen's estate elects the alternate valuation date, the stock will be valued at what amount in Joellen's gross estate? A) $6 million B) $5.6 million C) $5.7 million D) $5.5 million

If an estate elects the alternate valuation date and sells an asset before the six-month valuation date, the asset must be valued at its sale price, regardless of whether that price is more or less than the AVD value. LO 5.1.2

Which of the following statements accurately describes a reason for the suitability of an asset class in a qualified retirement plan portfolio? A) Common stocks offer capital appreciation and are relatively low risk. B) Mutual funds provide diversification and offer a competitive rate of return. C) Money market investments provide liquidity and a high real rate of return. D) Bonds are useful for funding future fixed obligations and offer protection against inflationary pressures.

In determining the suitability of an asset class in a retirement plan portfolio, mutual funds provide diversification and provide a comparable historical rate of return to that of equities (of course, depending on the investment objective and composition of the particular fund). LO 8.2.1

Which of the following statements regarding types of gifts is CORRECT? A) A gift of a future interest is not subject to gift tax. B) Indirect gifts, such as the payment of another's expenses, are not subject to the gift tax. C) The gift tax applies to incomplete gifts. D) A gift of a future interest is not eligible for the gift tax annual exclusion.

Indirect gifts, such as the payment of another's expenses, may be subject to the gift tax. The gift tax does not apply to incomplete gifts. The gift tax applies to gifts of future interests as well as to gifts of present interests. LO 2.2.1

Margo has made the following lifetime transfers to her spouse, Jim: She gave him a remainder interest in a parcel of real estate valued at $100,000. She gave him a life estate in her seaside cottage valued at $50,000 and did not make a QTIP election. She created an irrevocable trust that gave him a qualifying income interest and a general power of appointment over the trust asset and funded it with securities valued at $300,000. She created a QTIP trust, which she funded with securities valued at $200,000, gave him a qualifying income interest, and named her sister as the remainder beneficiary; she elected the marital deduction for the entire amount of the transfer. How much have Margo's lifetime transfers increased Jim's gross estate tax if Margo predeceases him, he does not remarry, and he retains the assets until death and they do not increase in value? A) $600,000 B) $400,000 C) $100,000 D) $650,000

Jim's gross estate will be increased by $600,000. All of the interests transferred will be included, except the life estate interest in the seaside cottage ($100,000 remainder interest, $300,000 marital trust interest, and $200,000 QTIP trust interest). The transfer of the three interests escaped gift taxation when made by Margo because they qualified for the unlimited marital deduction. Even though Jim has no control over who will receive the QTIP trust interest (as he does with the other two interests), the amount that escaped taxation because of the marital deduction will be part of his gross estate. Like the QTIP trust interest, Jim has no control over who will receive the seaside cottage at his death because his life estate rights end at his death, but the $50,000 value will not be included in his gross estate. This is because, unlike the QTIP trust, when Margo made the transfer of the life estate in the cottage, it was taxed as a gift due to it not qualifying for the marital deduction because it was a terminable interest and the QTIP election was not made. LO 2.3.1

Kim made the following gratuitous transfers in 2019: $20,000 certificate of deposit to her spouse $50,000 remainder interest in a trust to her two adult children $32,000 cash donation to her church $15,000 custodial account to each of her four grandchildren Kim's spouse consented to gift splitting for the current year. What is the amount of Kim's taxable gifts for 2020? A) $58,000 B) $5,000 C) $38,000 D) $25,000

Kim's taxable gift amount for the current tax year is $25,000. Only the gift of the remainder interest to her two adult children results in a taxable gift. Because Kim's spouse consented to gift splitting, Kim remains responsible for half of this taxable gift, or $25,000, which is fully taxable because it cannot be offset with the gift tax annual exclusion. Kim's gift of the $20,000 certificate of deposit to her spouse is reduced to a taxable gift of zero by the annual exclusion amount and the unlimited marital deduction ($20,000 - $15,000 = $5,000 - $5,000 = $0). Kim's gift of $32,000 in cash to her church is reduced to a taxable gift of zero by gift splitting, the annual exclusion amount, and the unlimited charitable deduction ($32,000 2 = $16,000 - $15,000 = $1,000 - $1,000 = $0). Finally, Kim's gift of $15,000 to custodial accounts for each of her four grandchildren also is reduced to a taxable gift of zero by gift splitting and the annual exclusion amount (for each child: $15,000 2 = $7,500 - $7,500 = $0). The other answer choices are incorrect because only gifts of present interests qualify for the annual exclusion. LO 2.2.2

Which type of assumed annual rate of return is frequently used in retirement planning calculations? A) Consumer Price Index (CPI) B) Black-Scholes model C) A flat annual return based on past performance D) Quantum simulation

Most planners use a flat annual rate of return based on past performance. The Black-Scholes model is a stock option valuation model. The Consumer Price Index (CPI) measures inflation. LO 8.3.1

Leilani made lifetime taxable gifts of $2.5 million. At her death in 2020, she was survived by her spouse and daughter and owned the following property interests: Sole ownership: Life insurance on her spouse's life with a death benefit of $5 million and a replacement cost of $100,000, with her children as the beneficiaries An investment portfolio worth $7 million A personal residence with a fair market value of $1 million Co-ownership: Property held with her spouse as joint tenants with right of survivorship (JTWROS) worth $4 million total Property held with her daughter as tenants in common, including a vacation condo valued at $750,000 (Leilani owns 80%) In her will, Leilani bequeaths the personal residence outright to her spouse and leaves the remainder of her probate estate to her daughter. Leilani had $200,000 of combined debts, funeral, and administrative expenses. Her estate paid $50,000 in state death taxes. Many years ago, she paid $210,000 in gift tax out of pocket. Which of the following amounts most closely approximates the net federal estate tax liability for Leilani's estate? Use the Unified Federal Estate and Gift Tax Rates table. A) $0 B) $414,900 C) $3,945,800 D) $4,300,000

Leilani's gross estate is $13,200,000 ($2.5 million in taxable gifts + $100,000 replacement cost of the life insurance on her spouse's life + $7 million in investments + $1 million personal residence + $2 million as her half of the JTWROS property + $600,000 as her share of the vacation condo). But, the residence and JTWROS property are eligible for the marital deduction, removing $3 million from her estate and thereby reducing it to a level where her applicable credit would eliminate taxes. The debts and expenses, death taxes, and gift taxes previously paid would further reduce the taxable estate. LO 5.2.2

Life insurance may be a suitable investment for all of the following retirement plans except A) profit-sharing plans. B) money purchase pension plans. C) defined benefit pension plans. D) SEP plans.

Life insurance cannot be held in any type of IRA, including SEPs. Qualified retirement plans and Section 403(b) plans can purchase life insurance if they comply with the incidental benefit rules. LO 8.2.2

Which of the following statements regarding prohibited transactions is NOT correct? A) One category of prohibited transactions involves the sale, lease, or exchange of any property between the plan and a party in interest. B) One category of prohibited transactions involves the investment in the sponsoring employer's stock or real property. C) One category of prohibited transactions bars a fiduciary from causing the plan to engage in a transaction if the fiduciary knows or should know that such a transaction constitutes a direct or indirect involvement between the plan and the parties in interest. D) The lending of money or other extension of credit between the plan and a party in interest is a prohibited transaction exemption.

Loans between the plan and a party in interest are prohibited transactions. LO 1.4.1

Mac, age 39, works for the SLH Company. He has a salary of $28,000 and a 401(k) there. Mac also has another job with AKH, Inc., making $65,000. SLH and AKH are not related. Mac is deferring $9,500 into the 401(k) at SLH. How much can he defer into the 401(k) at AKH? A) $10,000 B) $47,500 C) $57,000 D) $19,500

Mac can defer $10,000 into his 401(k) at AKH, Inc. The total he can contribute to employer retirement plans in 2020 is $19,500. Since he is already doing $9,500 at one employer, he is limited to $10,000 at the other. The fact that the two employers are unrelated does not matter for how much a worker can defer into the two plans. However, when it comes to unrelated organizations, each would have an independent annual additions limit. In other words, the law expects workers to know how much they are contributing to various employer retirement plans and to stay below the limit. The same would be true for related employers. However, unrelated employers are not held responsible for the employee or employer contributions to the other employers. LO 1.3.2

Which of the following is the easiest type of retirement plan for an employer to adopt? A) A custom plan B) A prototype plan C) An individually designed plan D) A Pension Benefit Guaranty Corporation (PBGC) plan

Master plans and prototype plans are easier to use than individually designed plans or custom plans because they are standardized plans approved as qualified in concept by the IRS. The PBGC is the governmental body that insures pension benefits; it is not a type of plan. LO 1.4.1

Sharon plans to retire next year and begin taking distributions from her traditional IRA. Her investment objectives are low risk, safety of principal, and liquidity. She is content with minimal rates of return. Which of the following investments is most suitable for her IRA? A) Dividend-paying common stock B) Corporate bonds C) Nondividend-paying common stock D) Money market instruments

Money market instruments provide the low risk, safety, and liquidity Sharon is seeking for her IRA. Corporate bonds and stocks do not provide safety of principal. LO 8.2.1

Which of the following statements regarding fee simple or absolute ownership of property is CORRECT? Fee simple ownership gives the owner only a lifetime right to use, possess or dispose of the property. Fee simple ownership gives the owner only a right at death to dispose of the property.

Neither I nor II is correct as both statements are false. A fee simple right gives the owner the right during lifetime and at death to dispose of the property. LO 1.3.1

Which of the following charitable remainder trusts can be revoked by the grantor after they are established? Charitable remainder annuity trusts (CRATs) Charitable remainder unitrusts (CRUTs)

Neither is correct. For a trust to qualify as a CRAT or a CRUT, the trust must be irrevocable upon inception.

John transferred a $2 million life insurance policy on his own life to an irrevocable trust nine years ago. The policy was paid up and had a gift tax value of $500,000. He paid gift tax of $198,000 out of pocket in the year of the gift. If John dies, how much is included in his gross estate? A) $500,000 B) $2,000,000 C) $198,000 D) $0

Nothing is included in John's gross estate because the policy on his own life was gifted more than three years prior to death. LO 5.2.1

Bernie and Tim, both age 53, are partners in a computer software consulting firm. They have 20 employees whose average age is 25 and average length of employment is three years. The firm is highly profitable and enjoys stable cash flows. Of the following retirement plan options, which is best suited to the partners' business? A) A stock bonus plan B) An eligible Section 457 plan C) Traditional defined benefit pension plan D) Section 403(b) plan

Of the listed plans, only a traditional defined benefit pension plan is available to Bernie and Tim's business. A Section 403(b) plan may only be adopted by a Section 501(c)(3) organization. An eligible Section 457 plan may only be adopted by a private, tax-exempt organization or a state or local governmental organization. A partnership cannot use a stock bonus plan because this form of business does not issue stock. Notice how subtlety the form of business was introduced in the question. The form of business is very important in selecting a retirement plan. LO 8.1.2

Which of the following statements regarding a rental property owned jointly with right of survivorship (JTWROS) is CORRECT? A gift is made when the name of the noncontributing joint owner (the donee) is added to the deed. The gift tax consequences with JTWROS are identical regardless of whether the tenants are spouses.

Only Statement 1 is correct. LO 1.3.1

Jack and Diane, a married couple, recently purchased a vacation home in their state of residence, which is a community property state. Assuming Jack dies, what percentage of the total ownership interest in the vacation home can Jack transfer under his will? A) 0% B) 50% C) 100% D) 25%

Only the portion owned by the decedent can be transferred by will. In a community property state, spouses own an equal undivided interest in all community property accumulated during their marriage. LO 1.3.3

Which of the following is not a broad category personal of financial planning goals? A) Financial nontax goals B) Nonfinancial goals C) Funding the collegiate education of a grandchild. D) Financial tax goals

Personal financial planning goals are typically thought of as tax versus non tax goals and financial and non financial goals. Funding collegiate education is not a broad category of a personal financial planning goal. It is, however, a type of financial non tax financial goal. LO 1.2.1

Which of the following statements regarding profit-sharing plans is NOT correct? A) Profit-sharing plans are types of qualified defined contribution plans. B) Profit-sharing plans are best suited for companies that have fluctuating cash flows. C) The maximum tax-deductible employer contribution to a profit-sharing plan is 25% of total (aggregate) eligible employee covered compensation. D) A company that adopts a profit-sharing plan must make contributions each year.

Profit-sharing plans are not required to make contributions each year. LO 3.1.1

Anne purchased property and titled it as joint tenancy with right of survivorship with her spouse. If she dies, how much of the property will be included in her gross estate? A) 50% B) 0% C) 25% D) 100%

Property held by spouses as joint tenancies with right of survivorship or as tenants by the entirety is not valued under the percentage-of-contribution formula. Instead, half is included in the estate of the first spouse to die, regardless of who made the contribution. LO 5.2.1

Each of the following are characteristics of the tenancy by the entirety form of property ownership except A) property that transfers by the right of survivorship. B) property that is owned only by spouses. C) property that is owned by more than two individuals. D) property that features creditor protection from the claims of each spouse's separate creditors, but not joint creditors.

Property owned as tenancy by the entirety may only be owned by husband and wife. LO 1.2.2

Which of the following situations would NOT constitute a transfer that comes within the gift tax statutes? A) Mary Sue creates a joint bank account for herself and her daughter, Rachel. Rachel has made no withdrawals from the account. B) Kai creates an irrevocable trust giving income for life to his spouse, Mina, and providing that at her death the corpus is to be distributed to their son, Jace. C) Bill purchases real property and has title conveyed to himself and to his brother, Joe, as joint tenants. D) Carla creates an irrevocable trust that provides that her daughter, Amy, is to receive income for life and that Carla's granddaughter, Hayley, is to receive the remainder at Amy's death.

Rachel has made no withdrawals from the account. A gift does not occur regarding the joint bank account until Rachel withdraws funds for her own benefit. LO 2.2.2

Which of the following statements regarding tenancy in common is CORRECT? A right of partition is inherent in a tenancy in common. Tenants in common must own equal fractional interests in the property.

Statement I is correct. Statement II is incorrect because tenants in common can own unequal fractional interests in the entire property. LO 1.3.2

Jeanine and Scott are married. Jeanine gifted $100,000 to their son and $100,000 to their daughter in 2020. Scott also gifted $50,000 to their son in the same year. They made no other gifts during the year. Jeanine and Scott elected to split the gifts on their gift tax returns. What is the total amount of taxable gifts made by Jeanine and Scott, respectively, for 2020? A) Jeanine: $185,000; Scott: $35,000. B) Jeanine: $95,000; Scott: $95,000. C) Jeanine: $110,000; Scott: $110,000. D) Jeanine: $170,000; Scott: $20,000.

Taxable gifts = $95,000 each ($250,000 − $60,000 annual exclusions) ÷ 2. LO 2.2.2

Billy owned a piece of undeveloped land (valued at $100,000) in San Jose, California, with his three brothers as tenants in common. If Billy dies, what value will be included in Billy's gross estate? A) $25,000 B) It cannot be determined from the information given. C) $100,000 D) $0

Tenants in common own an undivided interest in the property, but they may own unequal shares. These facts do not state what percentage of the property Billy owns. LO 1.3.2

When do the Chapter 14 rules of estate valuation generally apply? A) To discount the listed value of publicly held securities B) To determine the amount of the taxable gift in certain estate freeze transactions C) To determine the value of real property owned by a closely held business D) To arrive at the actuarial value of a life estate or remainder interest

The Chapter 14 rules of estate valuation generally apply in certain estate freeze transactions, such as corporate recapitalizations, between family members. These rules also apply in the valuation of interests in certain trusts, such as grantor retained income trusts (GRITs). LO 2.1.3

Max is the finance director for Bland Foods, Inc. He is trying to implement a new qualified retirement plan for the company. There are numerous federal guidelines with which the company must comply. Which of the following federal agencies is tasked with supervising the creation of new, qualified retirement plans? A) DOL B) PBGC C) ERISA D) IRS

The Internal Revenue Service (IRS) carries out the task of supervising the creation of new, qualified retirement plans. LO 1.1.1

Walter died with a gross estate of $18 million, which included an interest in a closely held business. His total estate tax is $2 million, and the estate tax attributable to the closely held business interest is $800,000. Assuming Walter's estate qualifies for the installment payment of estate taxes, what is the maximum amount of federal estate tax that may be deferred? A) $2,000,000 B) $800,000 C) $1,200,000 D) $400,000

The amount of federal estate tax that may be deferred is limited to the tax attributable to the value of the closely held business or businesses. LO 5.3.2

The estate tax applicable credit amount in 2020 effectively shelters what amount of taxable property transfers from the estate tax? A) $11,580,000 B) $15,000 C) $4,577,800 D) $1,000,000

The answer is $11,580,000. The 2020 applicable credit amount of $4,577,800 shelters up to $11,580,000 in taxable transfers at death (assuming no adjusted taxable gifts were made during the person's lifetime). LO 5.1.1

All of the following are considered qualified transfers for gift tax purposes EXCEPT A) Charles owns a residence with his sister as JTWROS. He transfers his ownership share of the residence to his sister, making her the sole property owner. B) Janet wrote a check for $30,000 to Boston College University to pay Ronnie's college tuition for the current school year. C) Lester paid $75,000 to Dr. Bonner for surgery he performed on his friend, David, last year. David is not Lester's dependent. D) Lucy transferred ownership of all of her shares in a mutual fund to her ex-spouse, Annie, because the written divorce decree in their divorce directed her to do so.

The answer is Charles owns a residence with his sister as JTWROS. He transfers his ownership share of the residence to his sister, making her the sole property owner. Charles's transfer of his ownership share of the residence to his sister was a gift, not a qualified transfer. The other transfers are all qualified transfers and not gifts. LO 2.1.2

Several of the steps involved in the process are Understanding the client's personal and financial circumstances Implement the recommendation(s) Identify and select goals Develop the recommendation(s) Which one of the following lists the sequence of these steps correctly?

The answer is I, III, IV, II. Understanding the client's personal and financial circumstances must be completed prior to identifying and selecting goals. From there, you can develop recommendation(s) and, with client agreement, implement recommendation(s). LO 1.3.3

Davis died in 2020 and was survived by his spouse and two children. At the time of his death, he owned the following property interests: Solely owned property valued at $6 million Property owned in joint tenancy with right of survivorship (JTWROS) with his spouse, with his share valued at $2 million Davis's will made no charitable bequests and provided that his entire probate estate go equally to his surviving children. Other pertinent facts include the following: Davis made $1 million in post-1976 taxable gifts. Davis's estate had $170,000 in allowable debts. Davis's estate had funeral expenses of $80,000. Davis's estate had administrative expenses of $100,000. Davis's estate paid $50,000 in state death taxes. Which of the following amounts most closely approximates Davis's federal estate tax base (also known as his tentative tax base, which is the amount with which he enters the estate tax table)? A) $8,400,000 B) $6,600,000 C) $7,150,000 D) $0

The answer is calculated as follows: Gross estate of $7 million ($6 million in solely owned property plus $1 million as his half of the JTWROS property), less $400,000 in deductions for expenses and debts, less $1 million for the marital deduction of the JTWROS property, equals a taxable estate of $5.6 million. To that, add $1 million for post-1976 gifts to arrive at $6.6 million. LO 5.2.2

Which of the following statements concerning income earned by spouses in a community property state is CORRECT? A) Income earned by each spouse prior to and after marriage is considered community property. B) Income earned by each spouse after marriage is considered community property only if it is commingled. C) Income earned by each spouse after marriage is considered community property. D) Income earned by each spouse is separate property.

The answer is income earned by each spouse after marriage is considered community property. Even though earned by only one spouse, such earnings are considered community property. LO 1.3.2

George is concerned that he will owe estate tax upon his death, and would like to make a transfer to reduce his gross estate. Which of the following transfer techniques would achieve his goal? A) Designating his children as payable on death beneficiaries of his bank accounts. B) Making annual gifts of an amount equal to, or in excess of, the annual exclusion. C) Retitling his residence with his children as JTWROS. D) Creating and funding a revocable trust for the benefit of his children.

The answer is making annual gifts of an amount equal to, or in excess of, the annual exclusion will reduce the value of the gross estate. LO 2.3.1

Which of the following transfers made by your client will be excluded from the client's total gifts in the year made? A) Made a $20,000 contribution to the Shriner's Hospital, which provides free medical care to children B) Made a $30,000 gift to the donor's spouse C) Paid a hospital $21,000 for medical services rendered to a friend D) Paid a family member $15,000 so she could go to school

The answer is paid a hospital $21,000 for medical services rendered to a friend as direct payment of medical expenses or tuition payments are exempt from gift taxes. LO 2.2.2

Which of the following statements about filing and payment of federal gift taxes are CORRECT? A) The federal gift tax return is IRS Form 706. B) A donor must file a gift tax return for a gift of a future interest only if its value exceeds the maximum annual exclusion. C) The donor of a gift is responsible for paying any federal gift tax due on that gift. D) A donor must file a gift tax return when establishing a QTIP trust only if a QTIP election is not made.

The answer is the donor of a gift is responsible for paying any federal gift tax due on that gift. LO 2.1.1

Which of the following statements regarding the community property system is CORRECT? A) Moving from a common law state to a community property law state automatically changes the nature of ownership of the property. B) Most states follow the community property system. C) The community property system generally assumes property acquired during the marriage belongs equally to both spouses. D) Moving from a community property state to a common law state automatically changes the nature of ownership of the property.

The community property system is followed in only a handful of states. Moving between community property and common law states does not automatically change ownership. LO 1.3.3

Saquon transferred the ownership of a whole life insurance policy he owned to his daughter last year. This policy had a face value of $1,500,000 and a cash value of $125,000. He also owns a universal life insurance policy with a death benefit of $250,000 and a cash value of $50,000. If Saquon died today, what amount of life insurance proceeds would be included in his gross estate for estate tax purposes? A) $1,750,000 B) $250,000 C) $1,500,000 D) $0

The death benefits of the whole life policy ($1,500,000) and the variable universal life policy ($250,000) would both be included in Saquon's gross estate. The death benefit from the whole life policy is included because Saquon transferred ownership of the policy within three years of his death. The death benefit from the variable universal life policy is included because Saquon owned the policy on his life at his date of death. LO 5.2.1

Generally, the due date of the federal estate tax return (Form 706) is A) 9 months after the date of death B) 12 months after the date of death. C) 6 months after the date of death. D) April 15 of the year following the date of death.

The federal estate tax return is due nine months after the decedent's date of death, although a six-month extension may be requested. LO 5.1.1

On January 1, 2020, Paul gifts a piece of land (basis of $100,000, FMV of $300,000) to his father. Which of the following statements is CORRECT? A) If Paul's father dies on December 1, 2019, and leaves the land to Paul, Paul will receive a stepped-up basis in the property. B) If Paul had sold the land to his father for $50,000 instead of gifting it, he would have had a deductible loss of $50,000. C) If Paul dies, the property will be included in his gross estate at the date-of-death value. D) Paul's father's basis in the property will be $100,000.

The gift basis to a donee is the carryover basis of the donor ($100,000). The boomerang rule of Section 1014(e) precludes Paul from receiving a stepped-up basis at his father's death if his father dies within one year of receiving the property from Paul. LO 2.1.1

What is the gift tax lifetime exemption amount for 2020? A) $15,000 B) $1,000,000 C) $11,580,000 D) $4,577,800

The gift tax lifetime exemption amount for 2020 is $11,580,000. LO 2.3.1

How are publicly traded stocks and bonds valued for gift tax purposes? A) The closing price on the date of the gift B) The bid price on the date of the gift C) The opening price on the date of the gift D) The average of the high and low trading price on the date of the gift

The gift tax value is the average of the high and low trading price on the date of the gift. LO 2.1.2

Andrea gifted property with a market value of $2 million to her daughter 18 months before she died. Andrea paid gift tax of $700,000 on the gift. At Andrea's death, the property had a market value of $2.5 million. What amount is included in Andrea's gross estate? A) $2,000,000 B) $2,500,000 C) $700,000 D) $500,000

The gifted property itself is not included in Andrea's gross estate, but the gift tax paid on the gift ($700,000) is included in Andrea's gross estate under the gross-up rule. The adjusted taxable gift from this transaction is not a part of her gross estate, but the adjusted taxable gifts are a portion of the total estate tax calculation further down in the process. The appreciation on the property after the gift is not subject to transfer taxation. LO 5.2.1

Which of the following statements correctly describes the gross-up rule? A) Gift taxes paid on gifts made within five years of the decedent's death must be included in the gross estate. B) Gift taxes paid on any gifts made during the decedent's lifetime must be included in the gross estate. C) The value of any gifts made within three years of the date of death must be included in the gross estate. D) Gift taxes paid on gifts made within three years of the date of death must be included in the gross estate.

The gross-up rule states that gift taxes paid on gifts made within three years of the decedent's death must be included in the gross estate. This is an example of being taxed on a tax. The deceased is estate taxed on the gift taxes paid out of pocket within three years of death (not the value of the gifts on which the gift taxes were paid). LO 5.2.1

The Jones Corporation has a profit-sharing plan with a 401(k) provision. The company matches dollar-for-dollar up to 5%. Pedro makes $150,000 and defers 5% into the 401(k) for 2020. The Jones Corporation has had a banner year and is considering a large contribution to the profit-sharing plan. What is the most that could be contributed to Pedro's profit-sharing account this year? A) $49,500 B) $19,500 C) $42,000 D) $57,000

The maximum allowed contribution for 2020 is $42,000. The section annual additions limit for 2020 is $57,000. However, Pedro has already contributed $7,500, and this amount has been matched. Thus, $15,000 has already gone toward the $57,000 annual additions limit for 2020. LO 1.3.2

Which of the following transfers is a taxable gift? A) John pays the University of South Texas $12,000 in college tuition for his 22-year-old niece. B) John pays $25,000 of medical expenses to a hospital for his invalid niece. C) John gives a $20,000 necklace to his fiancée 9 months before they are married. D) John donates $10,000 to his church.

The necklace is a taxable gift. Because John was not married at the time of the gift, he cannot take advantage of the marital deduction on gifts to his fiancée. Payments of tuition or medical expenses directly to a provider are NOT considered taxable gifts for gift tax purposes. The donation to the church is eligible for the gift tax charitable deduction. LO 2.2.2

Which of the following statements regarding a nonspouse designated beneficiary of an IRA is CORRECT? A) The nonspouse beneficiary may elect to distribute the IRA over the remaining life expectancy of the beneficiary commencing the year following the year of death, reduced by one for each subsequent year. B) The nonspouse beneficiary must take a lump-sum distribution in the year following the death of the participant-owner. C) The nonspouse beneficiary may rollover the IRA into his own IRA and defer distribution until attaining age 72. D) The nonspouse beneficiary must follow the five-year payout rule.

The nonspouse beneficiary may not rollover the IRA into his own IRA. While a lump-sum distribution or the five-year payout are options for the nonspouse beneficiary, they are not mandatory. LO 8.3.1

In 2020, Michael incurs substantial medical bills at the local clinic and Esteban pays $50,000 directly to the hospital in payment of Michael's medical expenses. What is the amount of Esteban's taxable gift as a result of this transaction? A) $85,000 B) $0 C) $30,000 D) $100,000

The payment of another person's medical expenses directly to the medical provider is a qualified transfer and is not considered a gift for gift tax purposes. LO 2.1.2

Which of the following is a tax-related financial goal? A) Minimizing nontax transfer costs B) Deferring the recognition of gain C) Maximizing benefits for a surviving spouse D) Maintaining a satisfactory standard of living

The remaining answer choices are nontax-related financial goals. LO 1.2.1

Which if the following is a nonfinancial goal? A) Preserving business value B) Asset protection C) Maximizing flexibility D) Minimizing nontax transfer costs

The remaining choices are financial goals. LO 1.1.1

Lisa has accumulated assets of $350,000 that she wishes to dedicate to her retirement. Inflation is anticipated to average 2% over the next 20 years. She plans on retiring in 15 years. What would be the value of her fund at retirement if Lisa can average a 5% after-tax rate of return on her accumulated assets? A) $467,035 B) $928,654 C) $520,082 D) $727,625

This is a future value calculation. PV = -$350,000 i = 5 (after-tax rate of return) n = 15 (years to retirement) FV = $727,625 LO 8.3.2

Last year, Nate established an irrevocable trust and funded it with his portfolio of income-producing stock valued at $440,000. The trust provides that the trustee is to pay Nate 6.5% of the initial value of the trust annually for a period of 15 years. After the 15-year term, the trustee is to pay the remaining assets in the trust to Nate's daughter, Karen. Which of the following is CORRECT regarding the gift tax implications of this trust arrangement? A) Nate's retained interest is not a qualified interest for IRC Chapter 14 purposes. B) Nate will have to pay gift tax only on the present value of the remainder interest. C) IRC Chapter 14 does not apply because this is an intrafamily transfer. D) Nate must file a federal gift tax return indicating that he has made a taxable gift of $440,000 to his daughter, Karen.

This is the correct answer because Nate will report a taxable gift of only the present value of the remainder interest, not the entire fair market value of the trust assets. Even though this is an intrafamily transfer, the usual valuation rules apply because the annuity interest retained by Nate is deemed to be a qualified interest. It is qualified because he will receive a fixed amount that will not fluctuate with the amount earned by the trust annually and cannot be manipulated—the trust is a grantor retained annuity trust (GRAT). LO 2.1.3

Which of the following situations does not require the filing of a federal gift tax return? A) A donor and spouse agree to split a present interest gift to one donee valued at more than the annual exclusion, but less than twice the annual exclusion amount. B) A donor makes a transfer to one donee of a present interest valued at less than the annual exclusion, but has used all of his applicable credit amount to offset the tax on prior gifts. C) A donor transfers to one donee a future interest valued at less than the annual exclusion amount. D) A donor makes a transfer to one donee of a present value for more than the annual exclusion, but has not used any applicable credit amount.

This is the correct answer because there is no requirement to file a federal gift tax return if the gift by a donor is of a present interest valued at less than the annual exclusion amount. The donee's applicable credit amount situation is irrelevant to whether a return must be filed. LO 2.1.1

Which of the following statements about the federal gift tax is CORRECT? A) The gift tax applies to all gratuitous transfers. B) The unlimited gift tax marital deduction has the effect of abolishing the terminable interest rule. C) Gift splitting means that spouses may elect to file a joint gift tax return. D) Taxable gifts for prior years must be added to taxable gifts in the current year to determine the tax bracket(s) applicable to the current year's taxable gifts.

This is the correct answer. Because the gift tax is cumulative (based on all taxable gifts since the inception of the gift tax in 1932), taxable gifts for prior years must be added to taxable gifts in the current year to determine the tax bracket(s) applicable to the current year's taxable gifts. LO 2.3.1

Which of the following is CORRECT regarding gift splitting between spouses for federal gift tax purposes? A) Only one spouse must be a U.S. citizen or resident before gift splitting is allowed. B) When spouses split gifts, one spouse "loans" the other spouse their annual exclusion amount. C) If the gift is of a present interest, more of the total gift value can be shielded from gift tax because each spouse may use their annual exclusion to reduce the taxable gift amount for their one-half of the gift. D) If one spouse consents to gifts made by the second spouse in a particular calendar year, but the second spouse does not reciprocate, gift splitting is allowed as long as the second spouse has consented in one or more prior calendar years.

This is the correct answer. It correctly states that if the gift is of a present interest, more of the total gift value can be shielded from gift tax because each spouse may use an annual exclusion to reduce the taxable gift amount for their one-half of the gift. Thus, up to twice the maximum annual exclusion amount can be sheltered with gift splitting, whereas only the amount up to the maximum annual exclusion amount may be sheltered without gift splitting. LO 2.2.1

Which of the following correctly describes the federal gift tax annual exclusion? A) It is available only to gifts made by married donors. B) It is the maximum amount of present interest gifts allowed per donee per year or the actual amount given to the donee, whichever is less. C) It allows a donor to completely avoid tax liability on a qualifying transfer of any amount. D) It applies to completed gifts of whole or partial interests and present or future interests.

This is the correct answer. It correctly states that the federal gift tax annual exclusion is the maximum amount of present interest gifts allowed per donee per year or the actual amount given to the donee, whichever is less. If the amount given is less than the maximum annual exclusion amount, the donor can only exclude the actual amount given. For example, if the amount given is $8,000, only $8,000 is excluded. The maximum amount is indexed annually for inflation, but only changes in $1,000 increments. LO 2.2.2

Jackie and Carmen are sisters who own real estate together. Jackie owns an undivided 35% interest in the property and Carmen owns an undivided 65% interest. Jackie and Carmen both have the right to sell their interest in the property or to leave their interest to anyone they choose under their wills. Which of the following describes this form of concurrent ownership? A) Tenancy by the entirety B) Joint tenancy with right of survivorship (JTWROS) C) Community property D) Tenancy in common

This property is held as a tenancy in common because each party owns an undivided interest in the property and the interests are unequal. The property is not held as JTWROS because each owner can dispose of her interest by will. Community property and tenancy by the entirety can only be owned by spouses. LO 1.3.2

Bob Dickson established a Uniform Transfers to Minors Act (UTMA) account for each of his 10 children in 2019. He funded each account with $580,000. Bob had made $200,000 of taxable gifts in prior years. The gift tax due on these prior gifts was paid by his gift tax credit amount. Bob did not make any other gifts to his children in 2020. Which of the following is the net federal gift tax due for the 2020 gifts? A) $232,000 B) $185,400 C) $67,525 D) $0

Transfers to an UTMA are eligible for an annual exclusion. Therefore, $565,000 of each transfer is taxable, for a total of $5,650,000. Bob's prior taxable gifts must be added to this amount to determine total taxable gifts of $5,850,000 ($5,650,000 + $200,000). This figure is taken to the gift tax rate table to compute a tax due of $2,285,800 ($4,850,000 × 40% = $1,940,000 + $345,800 [tax on $1 million]). From this amount, the tax on $200,000 (the prior taxable gifts) is deducted. The tax on $200,000 is $54,800. Therefore, $2,285,800 - $54,800 = $2,231,000. Because Bob has used $54,800 of his $4,577,800 gift tax credit amount on prior gifts, he has $4,523,000 of this credit left to apply to current gifts. Therefore, Bob's net federal gift tax due for the current transfers is $0 because he can use a portion of his remaining applicable credit amount. LO 2.3.1

Nigel's employer, Alpha, Inc., maintains a qualified defined benefit pension plan. There are 100 eligible employees working for Alpha, Inc. What is the minimum number of employees the retirement plan must cover to satisfy the 50/40 test? A) 100 B) 80 C) 40 D) 50

Under the 50/40 test, a defined benefit plan must cover the lesser of 50 employees or 40% of all eligible employees. In this case, the lesser of 50 employees or 40% of all eligible employees (100) is 40 employees. One way to remember the 50/40 test is the phrase people before percentages (50 people or 40%). Also, note that there are no qualifiers to the types of people. It is not 50 non-highly compensated people. It is just 50 individuals who work for the employer. LO 1.3.1

In 2020, Walter gave his 20-year-old son, Rufus, stock valued at $320,000. Walter's spouse, Frances, consented to split this gift with Walter for gift tax purposes. What amount of taxable gift must Walter report on his gift tax return for 2020? A) $145,000 B) $320,000 C) $305,000 D) $160,000

Walter must report a taxable gift of $145,000 on his 2020 gift tax return [($320,000 ÷ 2) − $15,000 annual exclusion]. The annual exclusion is considered after the gift is split between spouses. LO 2.2.2

Paulo and Francisca own a piece of property as tenants in common. Paulo owns an undivided 45% interest in the property, and Francisca owns an undivided 55% interest. The property has a fair market value of $1 million. If Francisca dies, what amount will be included in her gross estate for federal estate tax purposes? A) $500,000 B) $550,000 C) $1,000,000 D) $0

When property is owned as a tenancy in common, the amount included in the gross estate is based on the decedent's percentage of ownership. Francisca owns a 55% interest in the property, so her gross estate will include $550,000 ($1,000,000 × 55% = $550,000). LO 5.1.2

In which type of charitable trust does the income interest pass to a qualified charity and the remainder interest pass to one or more noncharitable beneficiaries? A) Charitable remainder annuity trust (CRAT) B) Charitable remainder unitrust (CRUT) C) Charitable lead trust (CLT) D) Pooled income fund (PIF)

With a charitable lead trust, income payments go to a charity and the remainder interest passes to noncharitable beneficiaries. With CRATs, CRUTs, and PIFs, the remainder interest passes to a charity. LO 2.2.1

Which of the following statements regarding the basis of inherited assets is CORRECT? When spouses own property as community property, both halves of the community property receive a stepped-up basis when the first spouse dies. When spouses own property as joint tenants with right of survivorship (JTWROS), 50% of the property receives a stepped-up basis when the first spouse dies. Inherited property that is income in respect of a decedent (IRD) receives a stepped-up basis when the decedent dies.

answer 1 and 2 Statement III is incorrect. Assets that are considered IRD do not receive a stepped-up basis at the decedent's death. LO 5.1.2

Which of the following statements regarding donor-advised funds is CORRECT? A donor-advised fund is an arrangement in which the donor makes a gift to charity and then makes future recommendations regarding who should receive grants or future monies from the charity. The major advantage of a donor-advised fund to the donor is the ability to name several charitable recipients. The costs to establish a donor-advised fund are high, as is the minimum required contribution.

answer 1 and 2 Statement III is incorrect. The costs to establish a donor-advised fund and the minimum required contribution are both low. LO 5.3.1

Which of the following transactions made in 2020 require the donor to file a gift tax return? The donor makes a gift of a future interest valued at $5,000 to his son. The donor and spouse use gift splitting and give their son $12,500 for the son's birthday. The donor transfers $15,000 to a revocable inter vivos trust for the son, who is both the income and remainder beneficiary. The donor and his spouse gift community property worth $14,500 to their daughter for her birthday.

answer 1 and 2 Statements I and II require filing. Statement III is not a completed gift, and Statement IV is a split gift of community property and is less than the annual exclusion; therefore, no filing is required. LO 2.1.1

Which of the following characteristics of the unified transfer tax system are common to both testamentary transfers and lifetime gifts? A single applicable credit amount that offsets taxes due A step-up in income tax cost basis when the transfer is made An annual exclusion per donee, per year from taxation A cumulative calculation using the same progressive tax rate schedule

answer 1 and 4 Statement II is false because only testamentary transfers receive a step-up in basis. Statement III is false because the annual exclusion applies only to lifetime gifts. LO 5.1.1

Mark wants to establish a grantor-retained trust that will provide him with fixed annual payments for the rest of his life. Which of the following trusts will meet his needs? GRAT GRUT

answer 1 only A GRAT provides the grantor with a fixed annuity payment. Statement II is incorrect because in a GRUT, the income interest payable to the grantor is in the form of a fixed percentage of the fair market value of trust assets, as revalued annually. LO 5.3.3

A gift of which of the following property interests will qualify for the gift tax annual exclusion? Present interest Future interest Remainder interest

answer 1 only Only a gift of a present interest qualifies for the annual exclusion. A remainder interest is an example of a future interest. LO 2.1.2

The Chapter 14 zero valuation rules focus on proper valuation of assets at the time of transfer for purposes of determining gift tax. Which of the following statements regarding the Chapter 14 valuation rules are CORRECT? An estate freeze involving the intrafamily transfer of corporate stock or partnership interests generally results in an immediate gift tax based on the entire value of the business held by the senior family member. In the case of buy-sell agreements, the Chapter 14 rules do not apply to transfers between non-family members.

answer 1 only Statement II is incorrect because in the case of buy-sell agreements, the Chapter 14 valuation rules apply to transfers between nonfamily members as well as transfers between family members. LO 2.1.2

Which of the following statements regarding adjusted taxable gifts (ATGs) is CORRECT? ATGs are taxable gifts made by the decedent after December 31, 1976. ATGs are included in the estate tax calculation at their date of death value. ATGs are included in the decedent's gross estate.

answer 1 only Statements II and III are incorrect because ATGs are added to the taxable estate (not the gross estate) at their date of gift value. All appreciation after the gift is made escapes transfer taxation for the original giver. LO 5.2.2

Which of the following statements regarding tenancies in common are CORRECT? Property is owned concurrently by 2 or more people. Each tenant's share is an undivided part of the entire property. There are no survivorship rights. The interests must be owned in equal percentages.

answer 1, 2 and 3 Statement IV is incorrect. Tenants in common may own unequal shares but only as an undivided interest in the entire property. LO 1.3.1

Which of the following statements regarding gift splitting are CORRECT? The annual gift tax exclusion allows spouses who consent to split their gifts to transfer up to $30,000 (for 2020) to any one person during any calendar year without gift tax liability, if the gifts are of a present interest. To qualify for gift splitting, a couple must be married at the time the gift is made. For gift tax purposes, spouses must file a joint income tax return to qualify for the gift splitting benefits. Both spouses must consent to the use of gift splitting and at least one gift tax return must be filed.

answer 1, 2, and 4 Spouses do not have to file a joint income tax return to elect gift splitting. The other statements are correct. LO 2.1.1

Holly read a newspaper article promoting lifetime gifting. Until reading this article, Holly had planned to leave all of her property through a testamentary transfer. What advice should you give Holly regarding the advantages of a lifetime gift over a testamentary transfer for a person who will owe estate taxes at death? The donee will usually take the donor's carryover basis. The gifted item is removed from the probate estate. A gift during life is less expensive than a testamentary transfer.

answer 2 and 3 A gift during life is less expensive than a testamentary transfer because of the annual exclusion and because any potential future appreciation is removed from the future transfer tax. Also, the gifted item is removed from the probate estate. Statement I is false because it is not an advantage. The donee does receive the donor's carryover basis, but it is not an advantage when compared to the stepped-up basis received with a testamentary transfer. LO 5.3.3

Two friends are planning to purchase a fishing lodge together. One friend will contribute 75% of the purchase price, and the other will contribute the rest. They prefer that the purchase of the lodge not generate any gift tax consequences for either of them. In addition, each friend has a will leaving all of his assets to his surviving spouse. They both want their interest in the property to pass under their will when they die. Which of the following forms of ownership would meet all of the friends' objectives? Joint tenancy with right of survivorship (JTWROS) Tenancy in common

answer 2 only JTWROS will not meet all of the friends' objectives. JTWROS is incorrect because a gift will result if one friend contributes more than 50% of the purchase price. In addition, JTWROS property will not pass under the friends' wills. Statement II (tenancy in common) is correct. Taking title as tenants in common will not result in a taxable gift and will allow the property to pass under the friends' wills. LO 1.3.2

In which of the following types of coownership does a deceased owner's share pass automatically to the surviving owner(s) at death? Tenancy in common Tenancy by the entirety Community property

answer 2 only Tenancy by the entirety is a type of joint tenancy with right of survivorship (JTWROS) that can occur only between spouses. The surviving spouse receives the deceased spouse's interest automatically by right of survivorship. Tenancy in common and community property do not pass by survivorship. LO 1.3.1

Which of the following statements regarding qualified transfers for tuition or medical expenses for gift tax purposes are CORRECT? The transfer is limited to the amount of the annual exclusion. The transfer reduces the annual exclusion dollar-for-dollar. The transfer must be paid directly to the medical provider or educational institution. The transfer can be made on behalf of anyone, without regard to the relationship of the donor, to the person benefitting from the gift.

answer 3 and 4 Statements I and II are incorrect. A qualified transfer for tuition or medical expenses is unlimited in amount and is independent of the annual exclusion or the person's relationship to the donor. Statements III and IV are correct. LO 2.2.1

Which of the following are examples of gift giving that are likely to result in favorable tax consequences? An advantage of giving property with a current value that is less than its basis (loss property) is that when the recipient sells the property the loss is available to offset any gains. Elderly taxpayers should give highly appreciated, low basis property in preference to cash. Making net gifts is a technique for clients who do not have very much in liquid assets and who want to make taxable gifts. The donee can depreciate depreciable property based on its value for gift tax purposes.

answer 3 only Statement I is incorrect; the double basis rule would not permit this favorable treatment. Statement II is incorrect; at the date of death, the property basis would be adjusted to fair market value. Statement III is correct. Statement IV is incorrect; the property would be depreciated on the basis of its adjusted basis, not its fair market value. LO 2.2.1

A grandfather decides to implement a systematic annual gifting program to make annual exclusion gifts to each of his 10 grandchildren every year. Which of the following statements regarding this gifting program are CORRECT? The grandfather will need to file a federal gift tax return each year. The program will require the grandfather to use a portion of his gift tax applicable credit amount. The program will allow the grandfather to reduce the size of his gross estate for estate tax purposes.

answer 3 only Statements I and II are incorrect. Because the grandfather's gifts will not exceed the annual exclusion amount, he will not be required to file annual gift tax returns and he will not use any of his gift tax applicable credit amount. Statement III is correct because any amounts gifted by the grandfather during his life will be excluded from his gross estate at death. LO 2.2.2

Tommy wishes to transfer his house valued at $60,000 to his son, Bob, in trust for Bob's lifetime, with the remainder to Bob's children. Tommy plans to occupy the house until his death. Which of the following statements are CORRECT? The gift to Bob is eligible for the annual exclusion. The gift of the remainder interest to Bob's children is eligible for the annual exclusion. Tommy has not made a gift. The transfer of the house is a future interest gift.

answer 4 only Neither the gift to Bob nor the gift of the remainder interest gift to Bob's children is eligible for the annual exclusion. A present interest gift that qualifies for the annual exclusion generally transfers an immediate right to possession or enjoyment of the property or property interest to the donee. In this case, the gift is one of a future interest, both to Bob and his children. LO 2.3.1

To qualify for the marital deduction, property must pass to the surviving spouse. How can property pass and still qualify for the deduction? By will By survivorship By intestacy By power of appointment

answer all All of these methods allow for property to pass and still qualify for the marital deduction. LO 5.2.1

The Chapter 14 valuation rules of the Internal Revenue Code apply to which of the following types of techniques between family members? Corporate recapitalizations Partnership capital freezes Grantor retained trusts Buy-sell agreements

answer all The Chapter 14 valuation rules apply to all of these techniques among family members. They also apply to buy-sell agreements among non-family members. LO 2.1.3

Erica owns a house that is not her personal residence and has a fair market value of $575,000. Erica's basis in the house is $400,000. She sells the house to her daughter for $450,000. Which of the following statements regarding this transaction is CORRECT? Erica has a taxable gain of $50,000. The property will not be included in Erica's gross estate when she dies.

answer both This is a bargain sale, and Erica has a taxable gain equal to the difference between the sales price and her basis. The property sold in a bargain sale is not included in the seller's gross estate; the taxable gift portion of the transaction is included as an adjusted taxable gift when calculating the seller's tentative tax base. LO 5.1.2

Which of the following decedent's estates might be eligible to elect the alternate valuation date (AVD)? Jeanine, whose gross estate is $50 million. She leaves her entire estate outright to her surviving spouse. Kim, whose gross estate is $6.5 million. She leaves her entire estate to charity.

answer neither Neither estate can elect the AVD because the AVD is not available when there is no estate tax due. Jeanine's entire estate qualifies for the marital deduction, and Kim's entire estate qualifies for the charitable deduction. LO 5.1.2

Ashley has a gross estate valued at $25 million when she dies. She is survived by her spouse, Roland, and their five children. Ashley had previously executed a will that incorporated bypass planning. Which of the following statements regarding Ashley's estate is CORRECT? Ashley's estate will owe significant estate tax. None of Ashley's estate qualifies for the marital deduction.

answer neither Statement I is incorrect because with bypass planning (the lifetime exemption amount to a B Trust and the remainder to the marital deduction), no estate tax is due because the entire estate is sheltered by either the estate tax exemption amount (the B Trust) or the marital deduction (the rest of the estate). The portion of the estate that exceeds the estate tax exemption amount qualifies for the marital deduction. LO 5.3.3

In which of the following situations is a holder's power to appoint property to himself considered to be a general power of appointment over the entire property for gift tax purposes? The holder's power is limited by an ascertainable standard so that the power is exercisable only for the holder's health, education, maintenance, or support (HEMS). The holder may exercise the power only with the consent of the grantor or a third party who has an interest that is adverse to the holder's. The right to exercise the power each year is limited to the greater of $5,000 or 5% of the total value of the property subject to the power.

answer none None of the statements describe situations in which a holder's power to appoint property to himself is considered a general power of appointment over the entire property. For gift tax purposes, the power of the holder to appoint property to himself is not a general power of appointment in any of these situations.

Which of the following statements regarding lifetime gifts are CORRECT? Annual exclusion gifts will escape gift taxation and will not be included in the donor's gross estate. Future appreciation in the value of gifted property will escape estate taxation in the donor's estate. Income from gift property will generally be taxed to the donee for income tax purposes. Generation-skipping transfer taxes do not apply to lifetime gifts.

answers 1, 2, and 3 Only Statement IV is incorrect; the generation-skipping transfer tax (GSTT) does apply to lifetime gifts. Statements I, II, and III are correct. LO 2.1.3

If Arthur and Tasha Bell begin making gifts to their three children in 2020, which of the following gifts would require the filing of a gift tax return for tax year 2020? Arthur gives each child $10,000, for a total of $30,000 in gifts. Arthur gives each child $10,000, for a total of $30,000 in gifts, and he and Tasha elect gift splitting. Tasha gives a future interest gift worth $5,000 to their daughter Danielle.

answers 2 and 3 Statements II and III are correct. A gift tax return must be filed whenever a married couple elects gift splitting and whenever a gift of a future interest is made. Statement I is incorrect; a gift tax return is not required in this case because no gift to any donee exceeds the annual exclusion amount. LO 2.1.1

Lionel Trane has made the following lifetime transfers: Gave his spouse a remainder interest worth $56,000 in a parcel of real estate after his death Funded a Section 2503(c) trust for the benefit of his daughter with $60,000 of common stock; his spouse did not split this gift. Paid his mother's medical bill to the community hospital in the amount of $15,000 Established a revocable trust for his only grandchild with $8,000 in cash Which of the following statements describe the tax impact of Lionel's lifetime transfers on subsequent lifetime transfers that Lionel may wish to make? The gift to his spouse will reduce the amount of future lifetime taxable transfers that he can make without having to pay the gift tax out of pocket. Establishing and funding the trust for his daughter will reduce the amount of future lifetime taxable transfers that he can make by the value of the gift, minus one annual exclusion, without having to pay the transfer tax out of pocket. Paying his mother's hospital bill will have no effect on subsequent lifetime transfers. Establishing the revocable trust will have no effect on subsequent lifetime transfers.

answers 2, 3 and 4 The gift to his spouse will have no effect because no part of it will be taxable. The entire amount will be covered by the unlimited marital deduction because the gift of a vested remainder is not a terminable interest. Also, since it is a future interest gift, the remainder interest is not entitled to an annual exclusion. The gift to the daughter's trust is entitled to an annual exclusion by the terms of Section 2503(c) even though it technically is not a present interest gift. Direct payment of medical expenses to the provider is exempt from gift tax. A revocable trust is revocable and therefore is not a completed gift.

Which of the following qualified plans can an S corporation implement? Profit-sharing plan Stock bonus plan Money purchase pension plan Employee stock ownership plan (ESOP)

answers all of them S corporations can establish stock bonus and ESOPs. LO 3.2.2

In the allocation of assets to determine the best portfolio composition for a qualified plan, what is a major factor to be considered? A) The administration requirements associated with the plan B) The type of plan and who bears the investment risk C) The maximum deductible contribution amount permitted under the plan D) The reputation and experience of the plan sponsor

A major factor to be considered in the asset allocation process of qualified plans is the type of plan (defined contribution or defined benefit) and who bears the investment risk (the employee or employer). Very broadly, a more conservative allocation is appropriate for a defined benefit plan than in the defined contribution approach. LO 8.2.1

Which of the following constitutes unrelated business taxable income (UBTI) to a qualified plan trust? A) Dividends from common stock purchased on margin B) Interest payments from corporate bonds C) Reinvested dividends from mutual funds D) Rents from real property investments

Unrelated business taxable income (UBTI) is gross income generated by a qualified plan trust that is not related to the function that is the basis for the trust's income tax exemption. In addition, the trust is generally prohibited from incurring debt. Therefore, dividends declared on common stock purchased on margin are treated as UBTI. LO 8.2.1

A planner may use a before-tax rate of return in making projections regarding the preretirement investment returns for all of the following assets except A) Roth IRA. B) Section 401(k) plan. C) mutual fund held outside a qualified retirement plan. D) traditional IRA.

Using a before-tax rate of return to project investment performance is appropriate for tax-advantaged assets, such as qualified plans and IRAs, because the earnings on these assets are not taxed each year as they accumulate. LO 8.3.1

Valerie earns $290,000 annually from XYZ Corporation. The company profit-sharing plan provides for a contribution of 25% of participant compensation. What is the amount of the company's contribution for Valerie for 2020? A) $230,000 B) $57,000 C) $100,000 D) $71,250

Valerie earns $290,000 annually in 2020. The plan provides for a contribution of 25% of participants' compensation. However, the maximum compensation that can be taken into consideration is $285,000. Twenty-five percent of $285,000 is $71,250. Finally, the maximum annual additions limit is $57,000, making the maximum company contribution for Valerie $57,000. LO 3.1.1

The Acme Corporation has six owners, ranging in age from 30 to 60 years old, and 25 rank-and-file employees. The owners want to adopt a qualified retirement plan that will allow them to maximize the contributions to the owners' accounts and minimize the contributions to the accounts of the rank-and-file employees. Which of the following plans would best meet the owners' needs? A) New comparability plan B) Age-based profit-sharing plan C) Section 401(k) plan D) Self-employed Keogh plan

A new comparability plan would allow the owners to divide the participants into two classes based on their compensation levels and allocate different contribution levels to the classes. An age-based profit-sharing plan wouldn't meet their objectives because the owners' ages are significantly different. Section 401(k) plans are subject to discrimination testing, and a self-employed Keogh plan is inappropriate because the owners aren't self-employed. LO 3.2.1

If the client's business objectives are to reduce income tax, reward executive employees, retain and recruit employees, and reduce employee turnover, which plan selection approach is the client using? A) A pension plan or a profit-sharing plan B) None of these C) Profit-sharing only approach D) Pension plan only approach

A pension plan or a profit-sharing plan can address each of these objectives. LO 8.1.1

In the administration of a qualified retirement plan, which of the following individuals is considered to be a fiduciary? A) A highly compensated employee who participates in the plan B) The marketing director of the plan sponsor C) A CPA who prepares the plan's Form 5500 D) A financial planner handling the investment of plan assets

A person or corporation is considered a fiduciary under ERISA if that person or entity renders investment advice or services to the plan for direct or indirect compensation. Clearly, the financial planner-investment manager is within this definition. LO 1.4.1

Which of the following statements regarding the characteristics or use of a profit-sharing plan is CORRECT? A) Profit-sharing plans require a fixed, mandatory, annual contribution by the employer to the plan. B) A company with a great number of older employees will find the implementation of a traditional profit-sharing plan to be the most beneficial for the older employee-participants. C) Profit-sharing plans are best suited for companies that experience fluctuating cash flow. D) The maximum tax-deductible employer contribution to a profit-sharing plan is 15% of covered payroll.

A profit-sharing plan does not require a mandatory, annual employer contribution. Therefore, they are best suited for companies that experience fluctuating cash flow and would like the flexibility of a discretionary (though substantial and recurring) contribution. The maximum tax-deductible contribution is 25% of covered payroll. Because there is less time to accrue a retirement benefit by retirement age, a profit-sharing plan is not the best choice for a plan with a great number older employee-participants. LO 3.1.1

Joe, age 52, has just started a consulting company. He currently employs six people, who range in age from 22 to 31 years old. Joe estimates the average employment period for his employees will be approximately three years and would like to implement a retirement plan that will favor older participants while including an appropriate vesting schedule. In addition, Joe would like the employees to bear the risk of investment performance within the plan. Which of the following plans is most appropriate for Joe's company? A) Cash balance pension plan B) SIMPLE 401(k) C) Target benefit pension plan D) SEP plan

A target benefit pension plan is likely most appropriate. It would permit Joe to favor older participants and allow for a vesting schedule. A cash balance pension plan does not favor older participants and provides employees with a guaranteed rate of return on investments (thus, not transferring the risk of investment performance to the employees). SEP plans and SIMPLE 401(k) plans both provide for 100% immediate vesting of employer contributions. LO 8.1.2

Jack, age 51, is the owner of an architectural firm with 23 employees, most of whom are younger than 40. The company's cash flow varies from year to year, depending on their contracts. Jack wants to implement a qualified plan that is easy for employees to understand and that is administratively cost-effective. He also wants a plan with an incentive feature by which an employee's account balance increases with company profits. Which of the following plans would be most appropriate for Jack's firm? A) Defined benefit pension plan B) Money purchase pension plan C) Traditional profit-sharing plan D) Section 403(b) plan

A traditional profit-sharing plan may be appropriate when an employer's profits, or cash flow, fluctuate from year to year; an employer wishes to implement a qualified plan with an incentive feature by which an employee's account balance increases with employer profits; or most employees are young (under age 50) and have substantial time to accumulate retirement savings, and the employees are, most likely, willing to accept a degree of investment risk in their individual accounts. Jack's company is not eligible to implement a Section 403(b) plan, as it is not a nonprofit organization. A defined benefit plan is not appropriate for the company's employee demographics, requires stable cash flows, and can be expensive to administer. Money purchase plans also require a mandatory contribution each year and do not provide the incentive feature regarding company profits. LO 3.1.1

A businessowner-client approaches a financial planner for advice on selecting a retirement plan for the business. What factors should guide the financial planner's recommendations? The owner's retirement savings need The owner's current age The amount of risk the client is comfortable assuming The financial stability of the business

All of the factors listed should be considered in selecting a retirement plan for the business. LO 8.1.1

Prohibited transactions are those that are not in the best interest of plan participants and include which of these? A loan between the plan and any party in interest The acquisition of employer securities or real property in excess of legal limits A transfer of plan assets to or use of plan assets for the benefit of a party in interest The sale, exchange, or lease of any property between the plan and a party in interest

All of the statements are prohibited transactions. Self-dealing is also a prohibited transaction. LO 1.4.1

An employee stock ownership plan (ESOP) is a defined contribution plan that may provide the employer with which of the following advantages? Increased corporate cash flow The ability to borrow money to purchase corporate stock A market for employer stock Financial resources to expand the business

All of these statements are ESOP advantages to the employer. LO 3.2.2

Grant, age 51, made an initial contribution of $10,000 to a Roth 401(k) in 2013. He made subsequent contributions of $6,000 annually for the next four years. In 2020, Grant took a $50,000 distribution from his Roth 401(k) to purchase a boat. Which of the following statements regarding this distribution is CORRECT? A) It is income tax free because it was made after five taxable years, and Grant is over age 50. B) It is income tax free because it was made after five taxable years from the date of initial contribution. C) It is partially taxable because Grant was not age 59½, disabled, the distribution was not made to a beneficiary or Grant's estate after his death, or used for a first-time home purchase. D) It is taxable because it was within 10 taxable years from the date of initial contribution.

Although Grant took the distribution after five taxable years from the date of initial contribution, he did not meet one of the other requirements for a qualified distribution (it must be made after the individual attained age 59½, the individual must be disabled, the distribution must be made to a beneficiary or estate of an individual on or after the individual's death, or it must be used for a first-time home purchase). In this case, the first $34,000 is counted against his contributions. Thus, there is no tax or penalty on $34,000. There are no conversions, so the rest of the distribution is earnings and thus subject to income tax and the 10% early distribution rules. LO 3.3.3

Which of the following retirement plans would be appropriate for a general partnership with stable cash flows? A) Age-weighted profit-sharing plan B) Employee stock ownership plan (ESOP) C) Section 403(b) plan (TSA) D) Stock bonus plan

An ESOP and stock bonus plan may only be established by an S or C corporation. A Section 403(b) plan (TSA) is only available to certain tax-exempt organizations. An age-weighted profit-sharing plan is the only plan appropriate for a general partnership from the choices given. LO LO 3.2.1

Which of the following statements regarding top-heavy plans is CORRECT? An accelerated vesting schedule is used when a defined benefit pension plan is top heavy. A qualified plan is considered top heavy if it provides more than 50% of its aggregate accrued benefits or account balances to key employees. Top-heavy defined benefit plans must provide a minimum benefit accrual of 2% per year of service for up to 10 years (20%) for all non-key employees. For a top-heavy plan, a key employee is an employee who owns more than 3% of the employer with compensation greater than $130,000 (2020).

Answer 1 and 3 An accelerated vesting schedule is used when a defined pension benefit plan is top heavy. A defined contribution plan always requires an accelerated vesting schedule. A qualified plan is considered top heavy if it provides more than 60% of its aggregate accrued benefits or account balances to key employees. A key employee is an employee who, at any time during the plan year, is the following: greater than a 5% owner; a greater than 1% owner with compensation greater than $150,000 (not indexed); or an officer of the employer with compensation greater than $185,000 in 2020. LO 1.3.1

Qualified retirement plans should do which of the following? They must meet specific vesting requirements. They have special tax advantages over nonqualified plans. They must provide definitely determinable benefits. They require an annual profit to allow funding for the plan.

Answer 1, 2, and 3 Qualified plans must meet specific vesting schedules. Qualified plans are preferred to nonqualified plans because of the special tax advantages enjoyed by qualified plans. Qualified retirement plans must offer definitely determinable benefits. An annual profit is not required for a qualified plan to be funded. LO 1.2.1

Which of the following are minimum coverage tests for qualified retirement plans? Nondiscrimination test Average benefits percentage test Ratio test Maximum compensation test

Answer 2 and 3 The two minimum coverage tests for qualified retirement plans are the average benefits percentage test and the ratio test. To be qualified, a retirement plan must meet at least one of these tests if the plan does not meet the percentage (safe harbor) test. LO 1.3.1

A client's employer has recently implemented a traditional Section 401(k) plan as part of its profit-sharing plan. Which of the following is CORRECT regarding the client's participation in the plan? A) The client is immediately vested in all elective deferrals and their accrued earnings. B) The client will not pay current federal income taxes on amounts distributed from the plan. C) The client is always immediately vested in all employer-matching contributions and their accrued benefits. D) The client will not pay Social Security (FICA) taxes on amounts paid into the plan.

Because a Section 401(k) plan is a qualified defined contribution plan, the employee is immediately vested (100%) in all elective deferrals and their accrued benefits. Such deferrals are, however, subject to FICA taxes. Vesting schedules may be used with employer-matching contributions. LO 3.1.1

Bernie is a participant in his employer's noncontributory employee stock ownership plan (ESOP). Two years ago, his employer contributed stock with a fair market value of $30,000 into Bernie's account. Bernie retired one year later and took distribution of the stock when its fair market value was $40,000. Two years after his retirement, Bernie sold the stock for $50,000. What is the appropriate tax treatment available to Bernie upon sale of the stock? A) $10,000 long-term capital gain B) $20,000 ordinary income C) $20,000 long-term capital gain D) $50,000 ordinary income

Employees are not taxed on stock in an ESOP until the stock is distributed. Upon distribution, the employee must pay ordinary income taxes on the fair market value of the stock when it was contributed to the plan on his behalf. Any net unrealized appreciation (NUA) at that time can be deferred until the stock's subsequent sale. Upon the sale, the NUA portion will be treated as long-term capital gain. Additionally, the growth of the stock subsequent to the distribution will receive long-term capital gain treatment because Bernie held the stock longer than one year after distribution. Therefore, the appropriate tax treatment available to Bernie upon sale of the stock is a $20,000 long-term capital gain. LO 3.2.2

The employee bears the investment risk in all of the following types of retirement plans except A) cash balance pension plans. B) money purchase pension plans. C) target benefit pension plans. D) traditional profit-sharing plans.

Employees bear the investment risk in defined contribution plans, and employers bear the investment risk in defined benefit plans. Cash balance pension plans are defined benefit plans; all the other answer choices are defined contribution plans. LO 8.2.1

A Section 401(k) plan does not have to satisfy the ADP and ACP tests if it meets one of the safe harbor provisions. All of the following statements about the safe harbor provisions are correct except A) mandatory employer contributions under the safe harbor provisions may be subject to a three-year cliff vesting requirement. B) an employer may satisfy the safe harbor provisions by making certain matching contributions for non-highly compensated employees who participate in the plan. C) a plan that satisfies the safe harbor provisions is not subject to the top-heavy rules. D) an employer may satisfy the safe harbor provisions by making non-elective contributions for all eligible employees.

Employees must be immediately 100% vested in mandatory employer contributions that are made under the safe harbor provisions. The mandatory contributions can take the form of either a non-elective contribution to all eligible employees or matching contributions to participating non-highly compensated employees who participate in the plan. LO 3.3.2

Paul estimates he will need $75,000 of annual income in today's dollars when he retires 10 years from now. He assumes a 3% annual rate of inflation, a 5% after-tax rate of return on his investments, and a 20-year retirement period. Using the serial payment approach, how much will Paul need to save in a single annual payment at the end of the first year to fund his retirement need? A) $100,794.00 B) $120,880.49 C) $115,124.27 D) $118,578.00

Paul will need to make a serial payment of $118,578.00. Here is how to calculate this type of problem using the regular TVM keystrokes: What will it take in ten years to have $75,000 of today's purchasing power? 75,000, +/-, PV 3, I 10, N FV Answer: $100,793.7285 What does Paul need to have in his account when he retires to produce a serial payment of $100,793.7285 at the beginning of the year for 20 years? +/-, +/-, PMT (This enters $100,793.7285 into the PMT as a positive value. The first +/- allows the calculator to accept the number. The second +/- makes it a positive number.) Shift, MAR (To get into the Begin mode.) 20, N 0, FV [(1.05 ÷ 1.03) - 1] × 100 = 1.9417, I PV Answer: $1,689,607.5595 What is $1,689,607.5595 in 10 years with inflation taken out of it? This step is critical because it prepares for the serial payment from today to retirement by removing the inflation from the dollar goal in 10 years. Serial payments account for inflation by adjusting the interest rate. Without expressing the FV in today's dollars, inflation will be double counted for the investment period. +/- $1,689,607.5595, FV (Only one +/- is needed here because the first +/- allows the calculator to accept the value and the PV was negative so after one +/- the new value will for FV will be positive.) 10, N 0, PMT 3, I PV Answer: -$1,257,226.7036 What is the first serial payment required at the end of the first year to have $1,257,226.7036 in 10 years? +/-, FV (This enters the result from aboe and changes the sing to positive $1,257,226.7036 Shift, MAR (To get in the End mode.) [(1.05 ÷ 1.03) - 1] × 100 = 1.9417, I 0, PV 10, N PMT Answer: $115,124.2738 This answer is in today's dollars, but the first payment will be made in a year, so... $115,124.27 X 1.03 = $118,578 LO 8.3.2

Several of the steps involved in the process are Develop the recommendation(s) Implement the recommendation(s) Identify and select goals Analyze potential alternate courses of action Which one of the following lists the sequence of these steps correctly?

The answer is III, IV, I, II. Understanding the client's personal and financial circumstances must be completed prior to identifying and selecting goals. From there, you can analyze the client's current course of action and potential alternative courses of action. After analysis, you can develop recommendation(s) and present them to the client. With client agreement, you can help implement recommendation(s) and, of course, monitor and update as needed. LO 1.1.1

Elizabeth and Tyler have been clients of yours for many years. In a recent meeting they share that in the last year three new grandchildren have joined the family. You recommend reviewing their beneficiary designations. This is an example of avoiding which of the following mistakes, pitfalls, or weaknesses? A) Improperly arranged life insurance B) Lack of estate liquidity C) Improper disposition of assets D) Failure to avoid ancillary probate

The answer is improperly arranged life insurance because it is the only answer choice which addresses changes in beneficiary designations. The remaining answer choices would also be good to avoid, but they do not involve beneficiary arrangements whereas life insurance certainly does. LO 1.2.1

Which of the following is not a mistake, pitfall, or weakness? A) Improper arrangement of life insurance B) Failure to minimize taxes C) Lack of estate liquidity D) Providing business planning

The answer is providing business planning because it is the only answer choice which would be a strength in an estate plan. The remaining choices are, in fact, mistakes, pitfalls, and weaknesses. LO 1.2.2

Alice participates in a qualified retirement plan at work. The plan provides Alice with life insurance. If Alice dies, which of the following statements correctly describes the income tax treatment of the life insurance death benefit paid to Alice's beneficiary? A) The pure insurance element of the death benefit is income tax free to the beneficiary. B) The beneficiary must pay income tax on the entire death benefit. C) The entire death benefit is income tax free to the beneficiary. D) The beneficiary must pay income tax plus a 10% penalty on the entire death benefit.

The beneficiary receives the pure insurance element of the death benefit income tax free. LO 8.2.2

Under a profit-sharing plan A) the company must make annual contributions. B) the employer bears investment risk. C) up to 25% of the plan's assets can be invested in the employer's stock. D) the company has flexibility regarding annual funding.

The company has funding flexibility. Pension plans can invest only up to 10% of plan assets in employer stock. Profit-sharing plans have no restrictions regarding investment in employer stock. The employer may deduct a contribution limited to only 25% of participating employees' covered compensation. They must make substantial and recurring employer contributions, or the IRS will remove the plan's qualified status. The employee bears investment risk. LO 3.1.1

What is the maximum elective deferral a participant can make to a Section 401(k) plan in 2020, assuming no catch-up provisions apply? A) $57,000. B) $19,500 C) $13,500 D) $6,000

The elective deferral limit for 2020 to a Section 401(k) is $19,500. The maximum annual additions limit is $57,000. LO LO 3.3.1

Jeanette uses the serial payment approach to calculate the amount she must save each year to accumulate her desired retirement income fund. She assumes an annual inflation rate of 4%, and an annual investment rate of return of 7%. If she determines that she must save $10,000 in the first year, how much must she save in the second year to meet her goal? A) $10,000 B) $10,288 C) $10,700 D) $10,400

Under the serial payment approach, the first-year savings amount is computed and then increased each year by the inflation rate. In this example, the amount in the second year is $10,000 × 1.04, or $10,400. LO 8.3.2

Window Washers, Inc., is establishing a profit-sharing plan using Social Security integration. The base contribution percentage for the profit-sharing plan will be 5%, and the owners have come to you with some questions about Social Security integration. Which one of the following statements is CORRECT? A) The excess contribution percentage for the plan could be as high as 26.25%. B) The permitted disparity for the plan is 3%. C) The excess contribution percentage for the plan could be as high as 10%. D) The permitted disparity for the plan is 5.7%.

The excess contribution percentage for the plan could be as high as 10%. The excess contribution percentage is the base contribution percentage plus the permitted disparity. The permitted disparity for the defined contribution plan is the lesser of the base benefit percentage and 5.7%. Thus, in this case, the permitted disparity is 5% and the maximum the excess benefit percentage could be would be 10%. LO 1.3.3

Jim, the president of East Dover Construction Company, has requested your advice in setting up a defined benefit pension plan for eligible employees in the company. Jim founded the company 17 years ago and now has 200 employees, most of whom are under 35 years of age. Due to the nature of the work and ongoing management difficulties, tenure among the employees has averaged under two years. Jim has just fired the managers who were creating problems, but turnover is likely to remain high due to ongoing morale problems. Jim's current salary is $300,000, and he wants the plan to provide him with annual retirement income of $100,000 per year. He expects to retire in 13 years, at age 64. Which of the following statements describes information you need to convey to Jim about factors that could affect the amount of his retirement benefit? A) If the plan investments outperform expectations, he will receive the largest allocation of the excess earnings. B) The excess integration method could be used to increase the amount of his plan benefit in retirement. C) Using a flat benefit formula will always provide Jim a larger benefit than would a unit benefit formula. D) One method of increasing the retirement benefit paid to him is by integrating the plan using the offset method.

The excess integration method increases the retirement benefit for compensation above the integration level. The offset integration method reduces the retirement benefit due to the receipt of Social Security benefits. With a defined benefit plan, excess investment performance reduces the subsequent employer contributions. A flat benefit formula versus a unit benefit formula would depend on the details of the flat benefit versus the unit benefit formula, so more information would be required to determine which would give a larger benefit. LO 1.3.3

In 2020, what is the maximum amount an employee under the age of 50 may contribute to a traditional Section 401(k) plan as an elective deferral? A) $26,000 B) The lesser of 100% of compensation or $57,000 annually C) $13,500 D) $19,500

The maximum amount of elective deferrals possible in 2020 to a traditional profit-sharing Section 401(k) plan for an individual under age 50 is $19,500 (it is $26,000 for an individual age 50 or older). The lesser amount of $13,500 applies to the SIMPLE 401(k) form. LO LO 3.3.1

Shawn, age 32, needs $10,000 for the purchase of a primary residence. She has no other source of funds at her disposal. Her Section 401(k) plan allows participant loans. The current value of Shawn's deferral account is $14,000, of which $9,500 is her aggregate vested balance. What is the maximum loan Shawn can take from the Section 401(k) plan? A) $10,000 B) $7,000 C) $14,000 D) $9,500

The maximum amount of loan Shawn can take is $9,500, which represents her aggregate vested balance. When the vested account balance is less than $20,000, loans up to $10,000 are available without regard to the half the vested balance rule. LO LO 3.3.1

Which of the following is a financial goal? A) Maximizing benefits for a surviving spouse B) Control of assets C) Meeting the needs of dependents D) The efficient transfer of assets at death

The other answer choices are nonfinancial goals. LO 1.2.1

Able Company is considering various types of qualified plans and seeks your advice. You are asked how a plan participant's benefits at retirement are determined in a defined benefit plan with a flat benefit formula that uses the offset method of integration. Which of these statements would best answer the company's question? A) The benefit paid from the employer plan is reduced by the amount the employer pays into Social Security on behalf of the employee. B) The benefit paid from the employer plan is based only on employee compensation above the Social Security wage base. C) The employee's Social Security benefit is reduced by a specific percentage of the retired employee's retirement plan benefit. D) The percentage of pay benefit specified by the plan is reduced by a specific percentage of the retired employee's Social Security benefit.

The percentage of pay benefit specified by the plan is reduced by a specific percentage of the retired employee's Social Security benefit. Offset integration reduces the defined benefit received due to the retiree also getting Social Security benefits. LO 1.3.3

What is the permitted disparity for a defined contribution plan with a current base contribution percentage of 6%? A) 11.7% B) 5.7% C) 12.0% D) 6.0%

The permitted disparity is the extra amount reached above the integration level. For an integrated defined contribution plan, the permitted disparity is the lesser of 5.7% and the base contribution percentage. Since the base benefit percentage is 6%, the permitted disparity is limited to 5.7%. That would mean the excess benefit percentage is 11.7% (6% + 5.7%). LO 1.3.3

Benjamin, who is unmarried, lives in Illinois and owns his home. He also owns a second home in Florida and has several liquid assets. Benjamin's goal is to leave all of his assets, including the two residences, to his brother, Daniel, while incurring the least amount of transfer tax and administration fees as possible. Benjamin is currently the sole owner of each property. As his planner, you recommend that he consider a revocable trust and title the Florida property in the name of the trust to avoid the administrative costs of an ancillary probate in Florida. Since Benjamin's goal is to reduce taxes, placing the property in joint tenancy with his brother would not be advisable since a gift tax liability would be incurred. Benjamin can also place his Illinois residence and his other assets in trust to avoid probate and reduce the estate administration fees and costs. This is an example of avoiding which of the following mistakes, pitfalls, or weaknesses? A) Failure to provide business planning B) Failure to minimize taxes and costs C) Lack of estate liquidity D) Failure to recommend necessary changes to a will

The recommendations made will reduce or avoid taxes and costs that would otherwise be incurred. LO 1.2.2

Which of the following is a tax goal related to transfer taxes? A) Obtaining a stepped-up basis B) Maximizing benefits for a surviving spouse C) Freezing the value of assets subject to tax D) Maintaining a satisfactory standard of living

The remaining choices are not tax goals related to transfer taxes. LO 1.2.1

Which of the following is a common mistake people make regarding estate planning? A) Adequate estate liquidity B) Proper titling of assets C) Recommending necessary changes to a will D) Improper disposition of assets

The remaining choices are the exact opposite of mistakes people make. LO 1.2.2

Which of the following is a common mistake people make regarding estate planning? A) Failure to give proper advice regarding funeral arrangements B) Proper titling of assets C) Recommending necessary changes to a will D) Adequate estate liquidity

The remaining choices are the exact opposite of mistakes people make. LO 1.2.2

A Section 401(k) plan allows plan participants the opportunity to defer taxation on a portion of regular salary or bonuses simply by electing to have such amounts contributed to the plan instead of receiving them in cash. Which of the following is a rule that applies to Section 401(k) plan elective deferrals? Section 401(k) plan elective deferrals are immediately 100% vested and cannot be forfeited. In-service withdrawals are to be made only if an individual has attained age 62. An extra nondiscrimination test called the actual deferral percentage test applies to elective deferral amounts.

answers 1 an 3 Statement II is incorrect. A Section 401(k) plan may allow in-service distributions before age 62. LO LO 3.3.1

Which of the following legal requirements apply to employee stock ownership plans (ESOPs)? ESOPs must permit participants who have reached age 55 and have at least 10 years of service the opportunity to diversify their accounts. ESOPs cannot be integrated with Social Security. An employer's deduction for ESOP contributions and amounts made to repay interest on an ESOP's debt cannot exceed 25% of the participants' payroll. The mandatory 20% income tax withholding requirement does not apply to distributions of employer stock from an ESOP.

amswers 1, 2, and 4 Options I and II correctly state the diversification rule and the rule that prohibits integrated ESOPs. There is no limit on amounts used to pay interest on ESOP debt. ESOP distributions of employer stock only are not subject to the 20% income tax withholding requirement. LO 3.2.2

In a Section 401(k) plan, which of the following must be considered in complying with the maximum annual additions limit? Employee contributions Catch-up contributions for an employee age 50 or older Dividends paid on employer stock held in the Section 401(k) plan Employer contributions

answer 1 and 4 Statement I is correct. Employee contributions are counted against the annual additions limit. Statement II is incorrect. Catch-up contributions for an employee age 50 or older are not counted against the annual additions limit. Statement III is incorrect. Earnings on plan investments are not taken into account when computing the maximum annual additions limit. Statement IV is correct. For 2020, the annual additions limit is the lesser of 100% of the employee's compensation, or $57,000. LO 1.3.2

Which of the following plans may be eligible for a 10-year forward averaging for tax purposes if a qualifying lump-sum distribution is made? Traditional profit-sharing plan Simplified employee pension (SEP) plan Individual retirement account (IRA) Section 403(b) tax-deferred annuity

answer 1 only Only lump-sum distributions from qualified plans may be eligible for 10-year forward averaging. SEP plans, IRAs, and tax-deferred annuities—also known as Section 403(b) plans—are not qualified plans and, therefore, are not eligible for 10-year forward averaging. Remember, to be eligible for 10-year forward averaging, a person must be born before Jan 2, 1936. Thus, only people working into their 80s can today qualify for 10-year forward averaging. Even then, they would have to take a lump-sum distribution to meet another rule to qualify for 10-year forward averaging. LO 3.1.1

Hardship withdrawals are only allowed from Section 401(k) plans if specifically stated in the plan document and typically for expenses such as vacation costs. medical expenses. college tuition costs. insurance premiums.

answer 2 and 3 Hardship withdrawals are typically allowed for medical expenses, college tuition and fees, to purchase a principal residence, burial expenses for spouse or dependents, and to prevent eviction from one's principal residence or foreclosure on the mortgage of such residence. LO LO 3.3.1

Which of the following penalties apply to prohibited transactions? A tax equal to 15% of the amount involved applies unless it can be demonstrated that the transaction satisfies ERISA's fiduciary standards. The transaction must be corrected and the plan placed in a financial position no worse than if the transaction had never occurred. Plan participants who engage in prohibited transactions are subject to income tax on a judicially determined amount. Transactions that continue uncorrected into subsequent years are subject to additional penalties.

answer 2 and 4 The law requires correction (i.e., undoing) of a prohibited transaction and restoring a plan to the position it would have been in, had the transaction never occurred. Ongoing transactions (e.g., loans, leases) create additional prohibited transactions in subsequent years (and additional penalties) until corrected. Options I and III are incorrect for the following reasons. Once the prohibited transaction has taken place, the 15% penalty cannot be waived for extenuating circumstances. Income tax consequences may or may not apply depending on the nature of the underlying prohibited transaction. Usually, the IRS (not the courts) determines the amount of tax involved. LO 1.4.1

Ryan wants to take a distribution from his SIMPLE 401(k) account balance from his previous employer and deposit it in an IRA at his local banking institution. Which of the following statements regarding his transfer is CORRECT? The distribution from the SIMPLE 401(k) plan is not subject to the mandatory 20% income tax withholding requirement. A direct transfer from Ryan's SIMPLE 401(k) to an IRA is not subject to the mandatory 20% income tax withholding requirement.

answer 2 only Statement I is incorrect. SIMPLE 401(k) plans are qualified plans and are subject to mandatory 20% income tax withholding for a distribution that is not a direct trustee-to-trustee transfer. When there is a direct transfer of a distribution from a qualified plan to an IRA, the mandatory 20% withholding rule does not apply. LO 3.3.3

Which of the following should a businessowner accomplish before considering the adoption of a retirement plan? Purchase personal and business liability insurance. Establish cash reserves sufficient to cover potential emergencies. Ensure the business has sufficient cash flow to support ongoing funding of the plan.

answer all of them A businessowner (or a person planning for his own retirement) should accomplish all of these objectives before considering the adoption of a retirement plan. LO 8.1.1

Which of the following statements regarding the anticipated effective income tax rate a planner should use for required retirement plan distributions is CORRECT? The projected rate should be based only on a blend of current federal and state marginal income tax rates. The projected rate should be based only on current federal marginal income tax rates. Precisely predicting future income taxes is not feasible. A planner should only use before-tax rate of return assumptions on retirement plan distributions.

answers 1 and 3 Before retirement, the planner may use a before-tax rate of return in the assumptions, particularly if tax-advantaged savings vehicles (such as a traditional or Roth IRA) are used in the planning process. However, at the time of either optional or required retirement plan distributions, the client's anticipated effective income tax rate is very important. This rate should be a blend of the client's federal and state marginal income tax rates but should be projected based only on current rates because precisely predicting future income tax rates is not feasible. LO 8.3.1

What may be the client's personal retirement plan objective if he selects a qualified defined benefit pension plan as the retirement plan for his company? An approaching retirement need A desire to maximize tax benefits to the business Selectively choose participants in the plan Maximize retirement contributions for older workers

answers 1, 2, and 4 Only Statement III is incorrect. A qualified pension plan is subject to nondiscrimination rules and the client would not be able to cover only select participants. LO 8.1.2

Which of the following statements regarding Section 401(k) plans is CORRECT? Employer contributions and their accrued earnings are always immediately 100% vested in the plan. A Section 401(k) plan is a qualified profit-sharing or stock bonus plan. The maximum pretax elective deferral for a participant age 35 is $19,500 in 2020. Elective deferrals in a cash or deferred arrangement (CODA) are not subject to FICA and FUTA taxes.

answers 2 and 3 Statements II and III are correct. Statement I is incorrect. Employee contributions and investment earnings are always 100% vested, not the employer contributions. Statement IV is incorrect. Elective deferrals in a CODA are subject to FICA and FUTA taxes. LO LO 3.3.1

Which of the following are basic provisions of an IRC Section 401(k) plan? Employee elective deferrals are exempt from income tax withholding and FICA and FUTA taxes. An employer's deduction for a vested contribution to a Section 401(k) plan cannot exceed 25% of covered payroll, which is not reduced by the employees' elective deferrals. A Section 401(k) plan cannot require as a condition of participation that an employee complete a period of service longer than one year. Employee elective deferrals may be made from salary or bonuses.

answers 2 and 4 Options II and IV correctly describe the 25% employer deduction limitation, eligibility requirement, and potential sources of employee elective deferrals for Section 401(k) plans. Option I is incorrect because employee elective deferrals (i.e., salary deferrals) are subject to FICA and FUTA taxes. Option III is incorrect because there can be a two-year period of service requirement if the participants are 100% immediately vested. LO LO 3.3.1

Which of the following can be used as a funding vehicle for tax-sheltered annuities (TSAs)? Stocks Bonds Annuities Regulated investment company shares held within a custodial account

answers 3 and 4 The primary investment vehicles for Section 403(b) plans are annuities or mutual funds held in a custodial account, although funds can also be used to purchase life insurance. Section 403(b) accounts can be invested in annuities or mutual funds held in a custodial account. Funds also can be used to purchase life insurance. Neither stocks nor bonds can be used as an investment for a Section 403(b) plan. LO 8.2.1

XYZ Inc. has 80 employees in the current year and is expected to employ the same number next year. They are considering the adoption of a retirement plan next year. The objectives are to use elective deferrals by employees with an appropriate match and immediate vesting. Which of the following plans would be appropriate? Traditional defined benefit pension plan Section 403(b) plan SIMPLE 401(k) SIMPLE IRA

answers 3 and 4 Traditional defined benefit pension plans do not permit employee contributions through elective deferrals. Section 403(b) plans are for 501(c)(3) tax-exempt organizations, and there is no indication that this is a tax-exempt organization. SIMPLE plans permit employee contributions through elective deferrals, matching, and immediate vesting. LO 8.1.2


Set pelajaran terkait

Chapter 26 Upper Respiratory Problems

View Set

Chapter 5: Individual Life Insurance Contract- Provisions and Options

View Set

Ch 11 Accounts Receivable, Notes Receivable, and Revenue

View Set

WGU484 multiple choice questions

View Set

Health and Life Insurance (Missed Questions)

View Set